PRITE 2022 - Assorted Pertinent Topics

Pataasin ang iyong marka sa homework at exams ngayon gamit ang Quizwiz!

The Culturally and Linguistically Appropriate Services (CLAS) federal standards for healthcare organizations require the organizations to:

"HAVE PT-RELATED MATERIALS AVAILABLE IN THE PT'S NATIVE LANGUAGES"

What is a pre-contemplative stage statement for tobacco cessation?

"HAVE YOU HEARD ABOUT THE HEALTH BENEFITS OF QUITTING?"

Pt's spouse calls the patient's psychiatrist to ask about changes in the patient's medication. Which of the following would be an appropriate initial response for the psychiatrist to make? (2x)

"INFORM THE SPOUSE THAT THE PATIENT'S CONSENT IS REQUIRED TO DISCUSS TREATMENT"

An effective role of C&L psychiatrist consulting on a manipulative, entitled patient?

"MODELING AN APPROACH OF APPEALING TO THE PATIENT'S ENTITLEMENT"

Reason lithium was hard to approve in US

"PHARMACEUTICAL COMPANIES WERE RELUCTANT TO PRODUCE A DRUG THAT THEY COULDN'T PATENT"

30 yo in therapy struggles with feelings of ambivalence about mom. Mom now has metastatic breast cancer. If the patient uses anticipation as a defense mechanism, she might:

"SET ASIDE A NIGHT OF THE WEEK TO HAVE DINNER WITH MOM TO DISCUSS MOM'S REACTION TO DIAGNOSIS"

Best response when patient asks you to lie about results to family

"TELL THE SPOUSE THAT THE PATIENT HAS NOT GIVEN PERMISSION TO DISCUSS PATIENT'S CARE"

"16 yo says she thinks she may be pregnant but does not want to involve her parents. What should guide the psychiatrist's actions?"

"THE PATIENT'S ABILITY TO GIVE CONSENT WILL VARY FROM STATE TO STATE TO STATE"

What is most important to evaluate "competence"? ;ϮxͿ

"WHAT DO YOU THINK WILL HAPPEN IF YOU DON'T UNDERTAKE THIS TREATMENT?"

Medical resident consults psychiatry because a 38 yo F pt refused dialysis secondary to ͞antisocial personality͟ and the resident wants you to convince her to stay for treatment. Your answer?

"WILL ASSESS PT'S COMPETENCY TO MAKE ONE'S OWN MEDICAL DECISION"

"Pt's dementia progressed to the point that the pt's driving is affected. In addition to talking to pt, the MD should do what (2x)"

"CONSULT STATE LAW ABOUT OBLIGATIONS FOR REPORTING THE PT'S CONDITION"

45 year old female with schizophrenia admitted for psychotic episode. Referring psychiatrist recommends that the resident give 100 mg of clozapine. The resident knows the starting dose should be lower and questions the dose. But the psychiatrist strongly repeats the recommendation. Next step?

"DISCUSS THE PSYCHIATRIST'S RECOMMENDATION WITH THE SUPERVISING ATTENDING"

"Pt qualifies for ECT, but does not want his doc to tell him the risks. He is a ͞chronic worrier͟ and fears hearing the risks will make him anxious, perhaps to the point of refusing treatment, which he believes he needs. What is the best response?"

"DOCUMENT THE PT'S KNOWING AND VOLUNTARY WAIVER AND PROCEED WITH TREATMENT"

"X yo pt presents to psych for treatment of ͞internet addiction.͟ Pt noticed a pattern of increasing time spent on the internet, need for better computer equipment, and feelings of anger and dysphoria when unable to access the Internet. Explain concepts of tolerance and withdrawal in this syndrome?"

BOTH TOLERANCE AND WITHDRAWAL TO INTERNET USE CAN DEVELOP

Accepting flowers from a paranoid patient to make patient feel relaxed

BOUNDARY CROSSING

Conducting a session after hours in a local coffee bar is an example of what

BOUNDARY VIOLATION

Difference between a boundary violation and boundary crossing (5x)

BOUNDARY VIOLATIONS ARE CHARACTERISTICALLY EXPLOITATIVE

This must be established in medical malpractice for plaintiff to prevail

BREACH OF DUTY CAUSING HARM

Routinely informing pts about confidentiality limits is an example of what ethical skill? (6x)

BUILDING ETHICAL SAFEGUARDS INTO WORK

Culture bound in industrialized countries per DSM IV

BULIMIA

A Japanese family who recently moved from Japan brings their 6 yo child for an evaluation. They express concerns that the child appears depressed, is reluctant to go to school, and is fearful that harm will befall the parents. During the evaluation it is noted that the child sleeps in the same room as the parents. Which of the following is the most likely explanation of this sleeping arrangement?

CULTURAL NORM

Number of new events in proportion to the population at risk for the event?

CUMULATIVE INCIDENCE

When evaluating for competence to proceed in a criminal matter, the psychiatric evaluator should focus on what aspect of the defendant? (2x)

CURRENT MENTAL FUNCTIONING

A patient w/ hx of psychosis is arrested for shoplifting and must be evaluated for ͞competency to stand trial.͟ What is relevant: ability to waive Miranda rights, intent at time of crime, understanding of illegality of behavior, working effectively with counsel, or decision making if actively psychotic

CAN THE PERSON WORK EFFECTIVELY WITH COUNSEL?

Type of payment involves insurance companies paying physicians a single standardized sum for patient care during a one-year period?

CAPITATION

Which is key component of Collaborative Care programs that integrate mental health services into primary care settings?

CARE MANAGEMENT

A psychiatrist working in a primary care clinic is contacted by the PCP for informal ͞curbside͟ consultation and provides recs without evaluating the pt. What is recommended to minimize liability?

CARE OF THE PATIENT REMAINS UNDER THE DIRECTION OF THE PCP WHO ALSO REMAINS IN CHARGE OF ORDERING MEDICATIONS OR ADDITIONAL SERVICES

Theorist that introduced the concepts of introversion and extroversion

CARL JUNG

Examining relationship between exposure to psychotropic during pregnancy and Epstein's anomaly between women with Epstein's anomaly and a group of women who had babies w/o this anomaly is an example of

CASE CONTROL STUDY

Research study: Daily cannabis use during adolescence in 200 adult participants with schizophrenia compared with use in 200 participants without a psychiatric diagnosis. Study type?

CASE CONTROL STUDY

Public mental health clinicians who follow patients through all phases of treatment

CASE MANAGERS

Examines the relationship between exposure and effect by comparing patients with effect to patients without effect, assessing for prior exposure. Type of study? (2x)

CASE-CONTROL

The rate of illicit drug usage in high school is highest among which groups in the USA

CAUCASIAN AMERICANS

Attribution processes are defined primarily as individual

CAUSAL EXPLANATIONS OF EVENTS AND PERSONAL EXPERIENCES

In a QI project which outcome indicator is feasible to measure and relevant to mental health

CHANGE IN DEPRESSION SCORE FROM ADMISSION TO DISCHARGE ON A RATING TOOL

Which of the following situational and environmental factors is most clearly associated w/ increased inpatient assaults? (6x)

CHANGE OF SHIFT

How to maintain professionalism with social media account?

CHECK THE ONLINE PROFILE REGULARLY FOR ACCURACY

29 yo schizophrenic, frequent ER visits and hospitalizations. Hallucinating, agitated, not violent or suicidal, cooperative. Ran out of meds a few days ago. Had been doing well on olanzapine for several months and was doing fairly well in a structured living environment. Denies command AH. Next intervention? (3x)

CONTACT CASE MANAGER TO VERIFY ENVIRONMENTAL SUPPORT

Insurance company tells the psychiatrist to switch to a generic drug. You know alternatives may be good too. What do you do?

CONTACT INSURANCE AND ASK FOR NON-FORMULARY APPROVAL

New instrument for PTSD does not ask about traumatic events. Raises which concern about validity?

CONTENT

Researcher is reviewing a questionnaire to screen for MDD, notices that the questionnaire has no Qs pertaining to depressed mood but has a few Qs pertaining to spending sprees. What form of validity is most affected? (2x)

CONTENT

The extent to which an instrument represents the construct being measured is referred to as what type of validity? (2x)

CONTENT

Managed care organization asks MD to d/c a pt who still requires hospitalization. MD should:

CONTINUE ADMISSION AS LONG AS MEDICALLY NECESSARY

Basic concept of community psychiatry

CONTINUITY OF CARE

Both mens rea and actus reus are required for which of the following?

CONVICTION OF A CRIME

According to theories of infant socialization, successful attachment most likely promotes survival through which of the following?

CREATING A BOND SO THAT THE ADULT WILL PROTECT THE INFANT FROM DANGER

Which of the following types of studies is best suited to determine prevalence? (2x)

CROSS SECTIONAL

Study type to measure if genetic test is accurate to make dx

CROSS-SECTIONAL

RCT where pt receives one med for 8 weeks, then 4 week break, then another med for 8 weeks. What kind of study?

CROSSOVER

Study design where subjects are their own pharmacologic control

CROSSOVER

"A clinical trial will assess the impact of two atypical antipsychotic meds on glucose intolerance. Half of the participants will start on drug 1 and then be treated with drug 2, while the other half will start on Drug 2 then drug 1. What type of experimental design?"

CROSSOVER DESIGN

After a pt assaults a roommate on a psych inpt unit the tx team decides to have pt arrested and transferred to jail. Decision is based on what

DUTY TO OTHER HOSPITALIZED PTS

"Legal responsibility to reach out to potential victim of pt's homicidal urges is called"

DUTY TO PROTECT

Small pilot study measures changes in MMSE scores to compare the efficacy of two meds for the treatment of Alzheimer's. Data says there's a difference between meds, but not statistically significant. What do you need to do to clarify?

ENROLL MORE SUBJECTS TO OBTAIN A LARGER SAMPLE SIZE

According to NIMH Epidemiologic Catchment Area Program's reports on ethnicity & prevention of mental disorders, the lifetime rates of APD are: (5x)

EQUAL AMONG ASIANS, HISPANICS, AFRICAN-AMERICANS, AND WHITES

Which of the following probabilities most accurately describes the likelihood of patients with schizophrenia committing homicide compared with the general population?

EQUAL LIKELIHOOD

Mental Health Parity and Addiction Equity Act of 2008 made which guidelines for substance use treatment

EQUALIZED VISIT LIMITS FOR BEHAVIORAL HEALTH AND MEDICAL CONDITIONS IN A SIMILAR CATEGORY OF COVERAGE

Which psychodynamic theorists posited that development occurs in sequential clearly defined stages and that each stage must be resolved for development to proceed smoothly?

ERIC ERIKSON

Who suggested that normal learning, learning of neurotic behavioral patterns, and unlearning of these behaviors through psychotherapy involves long-term functional and structural changes in brain through altered gene expression?

ERIC KANDEL

Pt consults a psychiatrist b/c former psychiatrist has retired & pt wishes to continue antidepressants. Pt is seeing a counselor for weekly psychotherapy and plans to continue. The psychiatrist's eval confirms the Dx of MDD and the psychiatrist feels that continuing the pt 's antidepressant is indicated. Next step? ;ϮxͿ

ESTABLISH A CLEAR UNDERSTANDING OF THE DIVISION OF RESPONSIBILITIES BETWEEN PSYCHIATRIST AND COUNSELOR

Power analysis is a statistical method used to:

ESTIMATE THE SAMPLE SIZE REQUIRED TO DETECT STATISTICAL EFFECT OF A DEFINED SIZE FOR TWO OR MORE PREDICTORS

Psych MD wishes to charge pts for appointments that they do not keep. According to code of ethics of APA, such a policy is: (3x)

ETHICAL IF PT IS SPECIFICALLY NOTIFIED IN ADVANCE

First psychiatrist to introduce term schizophrenia and describe primary/secondary symptoms?

EUGEN BLEULER

Pt getting psych exam to determine suitability for a job must be informed what

EXAM IS NOT CONFIDENTIAL

Largest contributor to waste in health care?

EXCESSIVE ADMINISTRATION

A required component of decisional capacity

EXPRESSED PREFERENCE

On-call resident evaluates a new patient with bad dementia. Resident realizes the patient was one of the resident's former professors and mentors. Resident is the only psychiatrist on the unit. Attending is called and tells the resident the hospital doesn't have a relevant policy. What's the optimal way to handle this?

FINISH THE EVALUATION AND PROVIDE ANY URGENT TREATMENT BUT INSIST ON A TRANSFER OF CARE AS SOON AS POSSIBLE

Which of the following is the most common method of completed suicide in female children and adolescents?

FIREARMS

Which quality improvement tool used to identify possible causes for an effect or problem and sort ideas into useful categories?

FISHBONE DIAGRAM

Essential element of wraparound treatment

FORMAL MENTAL HEALTH SERVICES AND INFORMAL COMMUNITY SUPPORT

Replaced use of hypnosis by Freud

FREE ASSOCIATION

Physicians can ask and counsel patients about guns based on what physician right?

FREEDOM OF SPEECH

What type of reduction strategy is needle exchange?

HARM

What type of intervention is naloxone education and access in areas with high area of opioid use disorder?

HARM REDUCTION STRATEGY

The four basic elements that must be proven in order to sustain a claim of malpractice against a physician include a duty of care owed to the patient, negligence, causation, and what?

HARM TO THE PATIENT

Resilient individuals who do well in developmental course through life despite being at high risk for negative outcomes are thought to be protected in adulthood most by

HAVING THE ABILITY TO FIND, USE, AND INTERNALIZE SOCIAL SUPPORTS

Suicidal 18 yo pt who lives with his parents is evaluated in the ED. Psychiatrist warns pt that if he does not sign in voluntarily for inpt tx, the psychiatrist will involuntarily hospitalize the pt. The pt consents to inpt care. The pt in this situation has not given valid informed consent because: (3x)

HE WAS COERCED INTO GIVING CONSENT

A test that detects 98% of the people who have a disease, but also provides a false positive result for many of the people without the disease, would have a: (2x)

HIGH SENSITIVITY AND LOW SPECIFICITY

Outcome of deinstitutionalization

HIGHER RATIO OF MENTALLY ILL PATIENTS IN PRISONS AS COMPARED TO HOSPITALS

Psychiatrist evaluates recent immigrant with complaints of ͞altered nerves.͟ Clinical interpreter from pt's cultural background explains that culturally the nervous system is believed to be altered by stressful life events. What cultural group does pt most likely belong to?

HISPANIC AMERICAN

Leading cause of death for 15-24 yo African American males?

HOMICIDE

Which of the following factors is most closely associated with child abuse and neglect?

HOUSEHOLD POVERTY

Piaget was interested in what

HOW A CHILD ARRIVES AT ANSWERS

"Piaget's contribution to development was a theory explaining what phenomenon (2x)"

HOW INDIVIDUALS LEARN AND UNDERSTAND THE WORLD

"In Vaillant's model of lifespan development, what is classified as a mature defense mechanism"

HUMOR

What somatoform d/o has male=female?

HYPOCHONDRIASIS

The rule that finds people not guilty by reason of insanity due to mental illness that impaired their knowledge of the nature, quality, and consequences of their actions or they were incapable of realizing that these acts were wrong is called what?

M'NAGHTEN RULE

In murder trial, defendant states ͞I think murder is wrong but killing a spy from Mars who is trying to steal the secret of life is right.͟ This poses a problem for what type of evaluation of criminal responsibility?

M'NAUGHTEN RULE

Best protection against malpractice lawsuit

MAINTAINING GOOD DOCTOR-PATIENT RELATIONSHIP

What distinguishes forensic from general psych evaluation? (2x)

MAINTENANCE OF CONFIDENTIALITY

"Winnicott's notion of capacity to be alone in presence of another person pertains to"

MAINTENANCE OF ONE'S IDENTITY AMONGST OTHERS

Testamentary capacity refers to person's ability to ;ϱxͿ

MAKE A WILL

Which application of competency requires the highest level of legal threshold?

MAKING FINANCIAL DECISIONS

The National Practitioner Data Bank contains reports about which of the following types of physician problems?

MALPRACTICE SETTLEMENTS WHERE NO WRONG WAS ADMITTED

According to C.G. Jung, anima refers to

MAN'S UNDERDEVELOPED FEMININITY

"What likely caused increased incarceration rates in the US between 1980 and 2010? (2x)"

MANDATORY MINIMUM SENTENCES FOR DRUG RELATED OFFENSES

According to Freud, what is a dream that is remembered on awakening

MANIFEST DREAM

Normal autism, symbiosis, differentiation, and practicing are among the developmental subphases of separation and individuation proposed by what theorist?

MARGARET MAHLER

29 yo complains about mistreatment from boyfriend. Proud of generous nature but complains how little she gets back. Therapist finds it hard to make her self-reflective about her role in this, gets frustrated and fatigued. Patient displays

MASOCHISM

Psychiatrist wants to talk to another physician and patient provides verbal consent but not written. What does HIPAA say

MAY DISCUSS THE CASE TO ADVANCE TREATMENT DECISION

"What psychiatric illness is the leading cause of disability in persons 15- 44 yo?"

MDD

Disorder carrying the greatest global burden of disease?

MDD

In a skewed distribution this is a measure of central tendency and moves furthest away from the mode in the direction of skewness

MEAN

The concept of lifetime prevalence refers to the number of persons:

MEASURED AT A POINT IN TIME, WHO HAD A D/O AT SOME POINT DURING THEIR LIVES

Measure of central tendency that might best be used to analyze an outcome measure with significantly skewed distribution of its values?

MEDIAN

Ten pts are given daily doses of a single antidepressant until their mood becomes euthymic. Eight pts respond to 1 mg, one pt responds to 5 mg, and one pt responds to 10 mg. What best reflects the skewed distribution of effective dose in this group of pts?

MEDIAN

The ABPN maintenance of certification program requires all psychiatrists to have what?

MEDICAL KNOWLEDGE

Psychiatrist sees the family of a pt who is in vegetative state. All staff, except one nurse, & all of the family, except one adult child,

MEET SEPARATELY WITH FAMILY AND STAFF TO EXPRESS ISSUES OF POLARIZATION

Main contributor to object relations theory

MELANIE KLEIN

The three components of informed consent include: the presentation of information, the voluntariness of consent on the part of pt, and (3x):

MENTAL COMPETENCE

"What is the main concept of the Mental Health Parity and Addiction Equity Act of 2008? (2x)"

MENTAL HEALTH INSURANCE BENEFITS CANNOT BE LESS FAVORABLE THAN MEDICAL OR SURGICAL INSURANCE BENEFITS

Pt in individual psychotx describes hatred for a "mean, unfair" boss. This time pt adds, "but actually, I have heard my boss talking to others in the office, and he seems pretty much ok - even his voice sounds so different from the way I hear him in my head!" According to Fonagy, pt is demonstrating which mental activity? (2x)

MENTALIZATION

Which psychodynamic theory emphasizes individuals ability to understand their own and others' internal states as explanation for behavior?

MENTALIZATION THEORY

Primary process thinking is characterized as

MENTATION FOCUSED ON DRIVES AND WISH FULFILLMENT

Results are combined from a number of studies of similar design. An overall estimate of the effect of a variable is made which incorporates the information provided by all the studies. The procedure is termed?

META-ANALYSIS

What relies on statistical review/interpretation of multiple study results?

META-ANALYSIS

A female child performs a dance for her mother and basks in the gleam of her mother's eyes. According to self-psychology, the child is having an experience of (2x)

MIRRORING

Which population has the highest prevalence of schizophrenia?

MONOZYGOTIC TWIN OF SCHIZOPHRENIC PT

Which of the following is a risk factor for suicide in a schizophrenic patient: female gender, low personal expectations, unawareness of symptoms, multiple admissions, younger age of onset?

MULTIPLE HOSPITAL ADMISSIONS

"Most common complaint in Southeast Asians who are diagnosed w/ mental disorder"

MULTIPLE SOMATIC SYMPTOMS

Intervention that exemplifies a wraparound, in-home, community- based treatment for youths?

MULTISYSTEMIC THERAPY

According to Freud, which psychosexual stage is associated with a primary erotic focus on the genital area and unconscious fantasies of sexual involvement with the opposite-sex parent? (2x)

PHALLIC

In what developmental period does a child see that he/she is a child of his/her parents & that parents have a relationship to each other not solely related to their roles as parents?

PHALLIC

"The case of ͞Little Hans͟ led Freud to develop a psychological theory of the formation of which of the following symptoms?"

PHOBIA

Factor causing decreased social networks in elderly:

PHYSICAL DISABILITY AND REDUCED MOBILITY

Agitated patient in ED. When to escalate to physical restraints?

PHYSICAL VIOLENCE

Evidence based treatment algorithm to maximize chances of successful consultation

PLAN A FOCUSED WORKSHOP FOLLOWED BY RETURN VISITS FOR FURTHER TRAINING AND PROBLEM-SOLVING SESSIONS

According to Kohlberg, moral judgments made by older school-age children are based largely on what

PLEASING THOSE IN AUTHORITY

The number of individuals with a d/o at a specific time (2x)

POINT PREVALENCE

Social psychological study of the view of self in some western cultures has delineated differences from the definition of self in many non-western cultures. What best describes the basis for positive self-esteem in those non-western cultures? (2x)

POSITIVE CONTRIBUTIONS TO THE GROUP AND HARMONIOUS RELATIONS

Cartesian dualism from the theories of René Descartes refers to

POTENTIAL OF HUMAN NATURE FOR BOTH GOOD AND EVIL

Method for estimating sample size required to detect statistical effects of defined size for variables with well-known variances is referred to as which of the following types of analysis (3x)?

POWER

What design measure is increased in a crossover study where study subjects serve as their own controls:

PRECISION

Pt able to recall his address after being prompted to do so. According to Sigmund Freud, this information is stored at what level of the topographic model?

PRECONSCIOUS

Pt wͬ hx of EtOH dependence BIB wife, who states that pt's long term use of ETOH has resulted in marital separation and persistent unemployment. Pr refuses to stop drinking. Psych MD asks pt about his perspective regarding his use of ETOH, pt responds, ͞It may be a problem.͟ Psych MD suggests Tx and pt says,͟I will think about it.͟ According to the stages of change model, which stage?

PRECONTEMPLATION

Post stroke, patient believes wife is a double who is impersonating her. Pt feels no emotional connection to wife. From learning theory, which processing error causes the delusional conviction?

PREDICTION

Animistic thinking is characteristic of which of Piaget's stages of cognitive development? (2x)

PREOPERATIONAL THOUGHT

To prove medical malpractice, the plaintiff must establish duty, dereliction, damages and direct causation by what part of proof

PREPONDERANCE OF EVIDENCE

What is associated with a better transition of care at end of life?

PRESENCE OF IDENTIFIED CARE PERSON

Pt w/ severe depression is interviewed for enrollment in an experimental trial. Pt has failed multiple medications and several courses of ECT. The pt is competent but clearly distressed at the time of consent for treatment. Next step?

PRESENT INFORMATION SEVERAL TIMES USING DIFFERENT METHODS AND FEEDBACK TESTING

Purpose of prescription drug monitoring programs

PREVENT DIVERSION OF DISPENSED PRESCRIPTION MEDICATIONS

Principal goal of wraparound services

PREVENT HOSPITALIZATION OR RESIDENTIAL PLACEMENTS

According to the USPSTF, which demographic group has the highest prevalence of depression?

PREVIOUSLY MARRIED

Raped 2 months earlier, the family insisted she not tell anyone. Understand stealing as: US Supreme Court Decision Washington v Harper 1990, the right to refuse treatment is limited for prison inmates because

PRISON SECURITY CONCERNS OUTWEIGH INDIVIDUAL AUTONOMY INTERESTS

Statistical concept of power is defined as the

PROBABILITY OF REJECTING NULL HYPOTHESIS WHEN IT SHOULD HAVE BEEN REJECTED

An attorney requests psychiatric consultation when his defendant wants the worst possible outcome from the trial because he feels he deserves the worst punishment possible. What direction should you take for this case?

PROBABLY MEETS CRITERIA FOR FINDING OF INCOMPETENCE TO PROCEED

One of the six core competencies per ACGME

PROFESSIONALISM

Which is a mandate of the Affordable Care Act designed to increase mental health service access?

PROHIBITION ON PRE-EXISTING CONDITION DENIALS DUE TO HISTORY OF MENTAL DISORDER

"When asked about a coworker after a recent conflict, a patient states ͞Ihey harbor no ill feelings toward him, but he truly hates me.͟ This is an example of which defense mechanism?"

PROJECTION

According to psychoanalytic theory, feelings of persecution are most reflective of which of the following defense mechanisms? (3x)

PROJECTION

Which defense mechanism most relevant to etiology of delusional disorders (like paranoia) according to psychodynamic theory (2x)

PROJECTION

A defense mechanism characterized by a self of object representation that is disavowed by placing it into someone else, and inducing the latter to unconsciously identify with the self/object representation (4x)

PROJECTIVE IDENTIFICATION

One group member alternates between being the ͞scapegoat͟ for the group or the ͞spokesman͟ for the group. These events are a group version of what?

PROJECTIVE IDENTIFICATION

Patient with abandonment issues asks therapist ͞what are your plans after residency?͟ Therapist begins dreading appointments and takes sick days on the days patient is scheduled. What defense mechanism?

PROJECTIVE IDENTIFICATION

Dementia study with 2 groups, one with history of lorazepam use, and one without, type of study design?

PROSPECTIVE COHORT

21 yo hospitalized for first psychotic episode. Parents want to be involved in care but pt does not want the team to speak to his parents due to his delusions. According to HIPAA, how much information can be shared?

PROVIDE INFORMATION GIVEN THE LACK OF CAPACITY AND LIKELY BENEFIT

Key role for psychiatrist in collaborative care model in the primary care setting

PROVIDES MEDICATION OR OTHER TREATMENT RECOMMENDATIONS TO THE PRIMARY CARE CLINICIAN FOR PATIENTS WHO ARE NOT IMPROVING

How to address people with physical and behavioral conditions in a collaborative approach?

PSYCHIATRIC CONSULTANT ADVISING THE PRIMARY CARE TEAM ON PATIENTS WHO ARE NOT SHOWING CLINICAL IMPROVEMENT

A lawsuit charges that a psychiatric facility prematurely discharged a patient who was not approved for further stay by the managed care company. The final legal responsibility generally falls to who (3x)

PSYCHIATRIST

Patient with MDD; psychiatrist prescribes antidepressant, psychologist provides interpersonal therapy. Who is responsible & accountable for what? (2x)

PSYCHIATRIST IS RESPONSIBLE & ACCOUNTABLE FOR ALL TREATMENT; PSYCHOLOGIST JUST FOR PSYCHOTHERAPY

Psychiatrist prescribes medication to a psychotherapy pt of another mental health professional. What is his liability in this treatment arrangement? (2x)

PSYCHIATRIST RETAINS FULL RESPONSIBILITY FOR PATIENT'S CARE

Radio listener sues psychiatrist after he sustains a fall and concussion after using a friend's alprazolam following a broadcast where psychiatrist mentions alprazolam is often prescribed for insomnia

PSYCHIATRIST WILL WIN BECAUSE A DOCTOR-PATIENT RELATIONSHIP WAS NOT ESTABLISHED

Which specialty has highest risk of suicide: Psychiatry, Ophthalmology, Family medicine, Emergency medicine, Cardiothoracic surgery

PSYCHIATRY

Exploration of transference needed for reparative emotional experience, what is this psychotherapeutic treatment called?

PSYCHOANALYTIC PSYCHOTHERAPY

Masturbation in adults as viewed by contemporary psychiatry is best described as

PSYCHOPATHOLOGICAL ONLY IF IT IS COMPULSIVE

Type of information patients cannot access even under HIPAA:

PSYCHOTHERAPY NOTES KEPT OUT OF THE GENERAL MEDICAL RECORD

Which is the most significant disadvantage of using a family member as interpreter for a patient who does not speak the psychiatrist's language?

PT MAY CENSOR COMMENTS TO KEEP INFO FROM THE FAMILY

Father is dying. Pt is not a drinker, but went from bar to bar drinking dad's favorite drink. Bars are similar to those dad used to go to. Best interpretation of this behavior

IDENTIFICATION WITH OR INCORPORATION OF PATIENT'S FATHER

Freud says depression is anger turned inward against self due to

IDENTIFICATION WITH THE LOST OBJECT

Specificity is what?

IDENTIFYING THE NUMBER OF PTS WITHOUT THE CONDITION WHO HAVE A NEGATIVE TEST RESULT

Freud says that boys resolve oedipal complex by

IDENTIFYING WITH FATHERS

A 23 yo pt began psychotherapy due to feelings of inadequacy and depression in the context of working in a coffee shop since graduating from college and deferring medical school acceptance. The pt has been having conflicts with parents over his ambivalence in becoming a physician. Which of the following is the most likely developmental task with which the pt is struggling? (2x)

IDENTITY

What limit to confidentiality exists for incarcerated patients?

IMMINENT PLANS TO ESCAPE OR CREATE DISORDER

What determines disability?

IMPAIRMENT AFFECTS ABILITY TO MEET PERSONAL, SOCIAL, OR OCCUPATIONAL NEEDS

Resident duty hour reforms have what consequences?

IMPROVED RESIDENT COGNITIVE PERFORMANCE

An effective consultation-liaison psychiatry program in a medical hospital will result in which of the following?

IMPROVED TREATMENT COMPLIANCE

Benefit of embedding psychiatrists in collaborative care teams in homeless shelters

IMPROVEMENTS IN COMMUNITY FUNCTIONING

What represents major goal of psychotherapy as conceptualized by self-psychology?

IMPROVING THE ABILITY TO USE THE APPROPRIATE AFFIRMING RESPONSES FROM OTHERS

According to Melanie Klein, a child who perceives the mother as an integrated person who is responsible for both good and bad experiences at the same time is said to be

IN THE DEPRESSIVE POSITION

Observational study with healthy pregnant women vs pregnant women on antidepressants, what is the study weakness?

INAPPROPRIATE CONTROL GROUP

This measure of disease frequency is most appropriate for determining the rate of new cases of a disorder in a given time period

INCIDENCE

A case-control study would be appropriate in answering an epidemiological question when the (3x):

INCIDENCE OF THE DISEASE IS LOW

"Most common allegation in malpractice suits against psychiatrist (3x)"

INCORRECT TREATMENT

Self-reporting tools used before assessment aid in what part of the initial psychiatric interview?

INCREASING THE EFFICIENCY OF THE OFFICE VISIT

"A preventative program for patients who exhibit attenuated psychotic symptoms and recent decline in functioning is what type of prevention program?"

INDICATED PRIMARY

APA positions on gun possession in psych patients

INDIVIDUALS WHO PRESENT AN INCREASED RISK OF VIOLENCE SHOULD BE BANNED FROM OWNING GUNS

10 yo pt reports he collects baseball cards. Child is excited because he is only one in his peer group who owns several cards. According to Erickson, what developmental tasks are most relevant for this child? (2x)

INDUSTRY VS INFERIORITY

According to Erikson, a child who strives to be competent by learning new skills, taking pride in results is which stage? (Also correlated with the Freudian stage of latency) (4x)

INDUSTRY VS INFERIORITY

Pt who recently delivered a baby brought to ER. Pt is agitated, labile, and demanding to leave because she said her baby is being poisoned. This condition is considered a psych emergency because of its association with what

INFANTICIDE

An accurate statement regarding assessment for cultural identity

INFLUENCED BY ONE'S DEGREE OF ACCULTURATION

Psychiatrist to eval pt with bipolar who is exhibiting threatening behavior while inpt on CCU for cardiac ischemia. Pt was incarcerated for assaulting an officer, disturbing the peace, and firearms violations. Pt verbally threatens to track down his ex-girlfriend and ͞teach her a lesson she'll never forget.͟ Psychiatrist has legal responsibility to do what

INFORM POLICE THAT PT IS IN POTENTIAL DANGER

What element besides potential for discomfort, bruising, infection, compensation for injury, and information about alternative treatments is necessary to cover the risks of a blood draw in an observational study?

INFORMATION FOR WHO TO CONTACT FOR A RESEARCH RELATED INJURY

Psychiatrist warns 24 yo pt that if pt does not sign in to a hospital voluntarily, the patient will be involuntarily committed. This approach violates the spirit of what

INFORMED CONSENT

Which of the following is most often cited by the literature as a significant barrier to adopting an electronic health record (EHR)?

INITIAL COST OF INTRODUCING A SYSTEM

27 yo uninsured pt was hospitalized in an intensive care unit after a near lethal OD which followed the ending of a long-term relationship. Pt is medically ready for discharge, but is ambivalent about follow-up psych care, noncommittal when asked about regretful feelings for surviving. Best approach?

INPATIENT HOSPITALIZATION

50 yo pt divorced and laid off from work presents to ED with SI with plan to OD on tylenol, lacks social support and impulsive history. Disposition?

INPATIENT HOSPITALIZATION

What distinguishes hypochondriasis from delusional d/o, somatic type?

INSIGHT

Major issue with maintaining severely mentally ill in community

INSUFFICIENT RESOURCES

Otto Kernberg's model for tx of patients with narcissistic personality disorder differs from that of Kohut in that in Kernberg's model, the goals of psychotherapy include what

INTEGRATING IDEALIZATION AND TRUST WITH RAGE AND CONTEMPT

24 yo Colombian-American is concerned about how to maintain his parents' cultural heritage after he marries a Caucasian. Which type of acculturative stress is he dealing with? (4x)

INTEGRATION

According to Erikson, person >65 yo with sense of satisfaction that life was productive/worthwhile has successfully managed psychosocial task of developing what (4x)

INTEGRITY

"According to Thomas Θ Chess' categories of temperament, the difficult child shows what characteristics?"

INTENSE EXPRESSIONS OF MOOD

Describe the psychiatric assertive community treatment case management model (2x)

INTERDISCIPLINARY TEAM, SERVICES IN SITU, HIGH STAFF/PATIENT RATIO, AND INTENSIVE OUTPATIENT RESEARCH

In the Institute of Medicine classification system developed to clarify different aspects of prevention, the category of ͞indicated interventions͟ refers to which of the following?

INTERVENTIONS THAT FOCUS ON HIGH-RISK INDIVIDUALS TO ENHANCE RESILIENCE AND PREVENT ONSET OF THE ILLNESS

Ethical standards dictate that the psychiatrist should do what in child custody evals?

INTERVIEW ALL PARTIES TO THE LITIGATION

Defense mechanism that involves internalizing the qualities of an object?

INTROJECTION

Person estimating usefulness of tx study will use number needed to treat statistic to represent number of pts who need to be treated to produce one additional good outcome beyond that obtainable w control for comparison condition. NNT formula?

INVERSE OF THE ABSOLUTE RISK REDUCTION

Example of a boundary violation:

INVESTING IN A PATIENT'S BUSINESS

31 yo postpartum F distressed by fantasies of smothering baby, tearfully describes feeling overwhelmed by the burden of the child care. She denies SI/HI, but describes feeling some evil external force taking her baby away. Requests outpt psych f/u & refuses voluntary admission, her husband says everything is fine. Next step?

INVOLUNTARY ADMISSION

According to the epidemiological Catchment Area Project, what condition is associated with higher rates of mood and anxiety symptoms compared to those with low back pain?

IRRITABLE BOWEL SYNDROME

"What characterizes the defendant's competency to stand trial in criminal proceedings?"

IS PRESUMED UNLESS THE ISSUE IS RAISED

Obsessive-compulsive symptoms are characterized by which defense mechanism?

ISOLATION AND UNDOING

Young man recounts how his father kicked his puppy to death, no emotion when telling the therapist this despite the therapist's upset response. Defense mech? (3x)

ISOLATION OF AFFECT

What made STAR*D trial more applicable to general public?

IT INCORPORATED PATIENT CHOICE INTO THE RANDOMIZATION STRATEGY

Appropriate substance abuse treatment in the clinical setting has been addressed in landmark legal cases. What is the consensus?

IT IS AMONG THE RIGHTS GRANTED AS IT IS CONSIDERED A PSYCHIATRIC DISORDER

Pt who is poor but talented asks if he can barter for services. You like the pt's product. What describes the nature of this proposal to accept goods in lieu of fees

IT IS NOT RECOMMENDED AS IT MAY COMPROMISE TREATMENT BOUNDARIES

Central principle of Goldwater Rule

IT IS UNETHICAL FOR PSYCHIATRISTS TO OFFER PROFESSIONAL OPINIONS ON PUBLIC FIGURES WITHOUT A FACE-TO-FACE EVALUATION

Patient with murderous impulses towards a neighbor gives the neighbor a bouquet of flowers. This is an example of what

REACTION FORMATION

"A benefit to unrestricted access to mental health information in the EHR for all specialties"

READMISSION RATES TEND TO BE LOWER IN HOSPITALS WITH UNRESTRICTED ACCESS TO THE EHR

An effectiveness study differs from an efficacy study by including what? (2x)

REAL WORLD CONDITIONS

24 yo pt comes to ED with wrist cuts. Reports having no intention to die but wanted to feel things. Admits to other self-harm behaviors when stressed. Admission to psych hospital would be most supported by what in the patient's history?

RECENT ALCOHOL ABUSE

Theory of kindling of depressive episodes is supported by what phenomena in some?

RECURRENT WITHOUT A STRESSOR

Psych unit staff wants initiative that decreases completed suicides inpatient. Which will have the greatest effect of decreased suicides while maintaining a therapeutic environment and minimizing staff burden?

REDESIGNING THE PHYSICAL SPACE TO ELIMINATE POTENTIAL ANCHOR POINTS FOR HANGING

Intensive case management is a program with elements of the assertive community treatment model, assertive outreach model, and case management model with a caseload of up to 20 patients. When compared to usual care, research demonstrates which desirable effects of intensive case management?

REDUCED LENGTH OF HOSPITALIZATION

Way sexual health changes in older individuals

REFRACTORY PERIOD FOR MEN INCREASES

Name the defense mechanism: a transient return to earlier behaviors characteristic of earlier stage of development in response to stress/conflict, despite overall maturation (2x)

REGRESSION

A method for making a prediction based on observable data in order to assess the valve of the one variable in relation to another is

REGRESSION ANALYSIS

What statistical measure should be used to determine how 2 independent variables affect the frequency of the occurrence rate of an event? (2x)

REGRESSION ANALYSIS

Odds ratio refers to what measure of association

RELATIVE RISK

What statistic will assess what percentage of side effects will be prevented with a new medication vs old one

RELATIVE RISK REDUCTION

This quality of a study indicates that the findings can be replicated

RELIABILITY

What DSM-IV diagnosis is likely for a 41 yo man with an increasing sense of conflict about changing his beliefs from work-oriented to faith-oriented. No past or present psych symptoms, stable vocational hx.

RELIGIOUS AND SPIRITUAL PROBLEM

47 yo F w/ increased preoccupation with questioning her lifelong affiliation with her church and has been spending time visiting other churches. Concerned she is losing her faith, feels guilty and anxious. No SI, MSE normal, states would never act against her moral beliefs. Dx? (3x)

RELIGIOUS OR SPIRITUAL PROBLEM

Best predictor of healthy aging in the elderly

REMAINING IN CONTACT WITH FRIENDS

What is primary role of psychiatric expert witness (4x)

RENDER OPINION BASED ON SCIENTIFIC KNOWLEDGE

Mother of ϴ yo wants ADHD eval, kids' parents are divorced, mom reports she has primary custody, will not sign ROI for child's dad, what is the next step?

REQUEST MOM PROVIDE COPY OF DIVORCE DECREE TO CONFIRM CUSTODY STATUS

Which of the following is the major negative aspect of crossover designs for research studies? (3x)

RESIDUAL EFFECTS

In community-dwelling older adults, greater spirituality is associated with higher: income, resilience, education, rates of alcohol use, rates of cognitive decline

RESILIENCE

Personality characteristic as predictor of successful aging: rigidity, resilience, dependency, perfectionism, emotional reactivity

RESILIENCE

As part of a comprehensive pain management team for pt w/ hx of narcotic abuse, a consulting psychiatrist may appropriately be asked to do which of the following? (2x)

RESOLVE CONFLICTS BETWEEN THE PATIENT AND THE TREATMENT TEAM

The Belmont Report states that the underlying principle of informed consent is

RESPECT FOR PERSONS

Physician involvement prohibited in which aspect of the death penalty without a commutation order

RESTORATION OF COMPETENCE TO BE EXECUTED

Situations where patients can be forced antipsychotics include emergencies, treating symptoms threatening others' or selfΖs safety and what

RESTORING A CRIMINAL DEFENDANT TO STAND TRIAL

Which ego defense is seen when an adolescent belittles parents in order to defend against regressive pull toward childhood?

REVERSAL OF AFFECT

Patient with schizophrenia and afib admitted to inpatient unit. Overnight resident on call forgets to order patient's home warfarin, and patient gets a stroke. What is the next best step to perform a root cause analysis of this adverse event?

REVIEW OF THE PATIENT'S CLINICAL DOCUMENTATION

Name of culture-bound syndrome in Caribbean and states bordering Gulf of Mexico ʹ anxiety, GI distress, weakness, fear of being poisoned, attributed to witchcraft

ROOTWORK

"Guardian of a pt with intellectual disability with medication-refractory epilepsy insists that the pt should have epilepsy surgery. Neurosurgeon believes the seizure focus can be resected, but there's a >50% chance of RUE paralysis which outweighs potential benefit from freedom from seizures. Guardian insists on surgery. Which describes physician responsibility in this case?"

SHOULD PERFORM THE SURGERY ONLY UNDER COURT ORDER

Prevalence of a disease increases with either a slow or rapid course?

SLOW

What most accurately describes scientific understanding of the concept of race?

SOCIOPOLITICAL DESIGNATION ASSIGNING INDIVIDUALS TO A PARTICULAR GROUP THAT HAS MEANING DERIVED FROM PREVAILING SOCIETAL ATTITUDES

Pt has psychotherapist. Pt referred to psychiatrist who prescribes medication. Psychiatrist's responsibility to pt

TO FOLLOW UP BY SEEING PT AT APPROPRIATE SCHEDULED INTERVALS AND BY COMMUNICATING REGULARLY WITH PSYCHOLOGIST ABOUT PT'S CLINICAL STATUS AND TREATMENT PLAN

#1 preventable cause of death in USA

TOBACCO USE DISORDER

This model divides the mind into conscious, preconscious, and unconscious? (2x)

TOPOGRAPHICAL

Integrated setting with primary care and mental health, which would be population based techniques to provide collaborative care?

TRACKING CLINIC PATIENTS IN A REGISTRY

What intervention can a hospital system employ that would most optimally protect against EMR data breaches?

TRAIN EMPLOYEES ON HOW TO RECOGNIZE POTENTIAL THREATS

During therapy, therapist coughs while patient is speaking. Pt angrily states, If you disagree with me just tell me, but don't be a passive aggressive jerk!" Therapist interprets that pt is consistently treating the therapist like an aggressor. Which best describes this therapy modality?

TRANSFERENCE FOCUSED PSYCHOTHERAPY

Pt's insurance refuses to pay for continued hospitalization for a depressed patient who is acutely suicidal. The psychiatrist remains concerned about SI and does not discharge the patient. This best illustrates the ethical principle in managed care of responsibility to: treat, appeal, disclose, respect autonomy, cooperate with utilization review

TREAT

Constitutional and common law dictate that mental health care provided to prisoners be at or above the level of services that should be available to them if they were in the community. The "necessaries" doctrine is justified by what principle? ;ϯxͿ

TREATMENT MUST BE PROVIDED TO THOSE WHO ARE PREVENTED FROM SEEKING THEIR OWN MEDICAL CARE

Fitness for duty evaluations done by administrative psychiatrists are not bound by the usual standards of confidentiality (true or false)?

TRUE

Medicare system part B is an optional purchasable benefit for senior citizens; true or false?

TRUE

Son of 70 yo pt calls psych MD to report his mom has become depressed in the last 6 months, but pt denies being sad and refuses to see psych MD. Son mentions that he will bring mother to the appointment under the pretense of having her back pain checked. Most appropriate response from psych MD would what

TRY TO DISSUADE THE SON FROM USING DECEPTION

Couple in therapy, review argument at a family reunion, the husband told his sister-in-law that his wife was superior to her in every way, the wife expressed disapproval, the husband became quiet and later fell down a flight of stairs. What defense mechanism is this?

TURNING AGAINST SELF

Which study best addresses the relative influences of heritability and environment?

TWIN ADOPTION STUDIES

Study reports difference that turns out to be by chance, what error is this? (2x)

TYPE I ERROR

Several states established that the verdict guilty but mentally ill to be an option when a defendant pleads not guilty by reason of insanity because

WHEN COMPARED TO GUILTY, GUILTY BUT MENTALLY ILL IS INTENDED TO ENSURE ACCESS TO TREATMENT BUT IS ESSENTIALLY SIMILAR

Most appropriate time to discuss advance directives with a pt (2x)

WHEN THE PT IS COMPETENT

When should a schizophrenic admitted to a medical hospital for evaluation of chest pain have a formal assessment of decision-making capacity?

WHEN THERE IS REASON TO BELIEVE THAT THE PATIENT LACKS THE ABILITY TO UNDERSTAND, APPRECIATE, OR REASON LOGICALLY WITH THE INFO RELEVANT TO HEALTHCARE DECISIONS

The most relevant issue for a geriatric or forensic psychiatrist in cases were the finances of an elderly are in dispute between partner and children is

WHETHER THE ELDERLY HAS A MENTAL DISORDER AND WHETHER IT IS ADEQUATELY TREATED

When documenting suicide risk assessment, key strategy is to discuss what

WHY THE PSYCHIATRIST REJECTED ALTERNATIVE WAYS OF RESPONDING

Menninger's triangle of insight provides a framework for identifying links between the patient's current interpersonal relationships, early- life interactions with family members or other important caregivers, and the transferential relationship with the therapist. What is this process?

WORKING THROUGH

What is the legal process that asks the court to immediately evaluate if a pt has been hospitalized w/o due process of law? (2x)

WRIT OF HABEAS CORPUS

A resident is discharging a patient and notes a significant omission in the resident's last, handwritten progress note. Which of the following is the best course of action?

WRITE AN ADDENDUM TO THE PROGRESS NOTE WITH THE CURRENT DATE

Which mental illness carries the highest economic burden of chronic disability relative to its incidence and prevalence?

SCHIZOPHRENIA

Example of secondary prevention

SCREENING FOR DEPRESSION

Female pt in twice-weekly psychodynamic psychotx w/ male therapist for the last year begins to discuss sexual fantasies about the therapist. The therapist feels flattered by the sexual interest of this patient and notes his own feelings of attraction to her. Most appropriate next step for therapist? (5x)

SEEK CONSULTATION WITH A COLLEAGUE

Pt with depression recounts to the psychiatrist events of the last week and describes many unpleasant experiences, concerns about the sadness of some friends, and a reluctance to read the newspaper because ͞all the news is bad.͟ In cognitive psychology, which of the following formulations best accounts for this patient's presentation?

SELECTIVE ATTENTION BIAS

The fundamental developmental need of all persons for mirroring, validation, and affirmation is central to which psychoanalytic theory?

SELF PSYCHOLOGY

What term describes the role that others perform for the individual in regard to mirroring, idealizing, and twinship needs?

SELF-OBJECT

Psychodynamic therapy was derived from which school of psychoanalytic theory?

SELF-PSYCHOLOGY

A statistical term that refers to the proportion of pts with the condition in question that a test accurately detects

SENSITIVITY

What measure should be maximized in a screening test to miss the fewest individuals with an illness

SENSITIVITY

PHQ-2 score cut off is 3. If cut off is lowered to 2, what would happen?

SENSITIVITY WOULD INCREASE

What sexual behavior is most likely preserved in older patients?

SEXUAL TOUCHING

As conceptualized by self-psychology, which of the following is the most problematic reaction a patient can have when attempting to fulfill a self-object need?

SHAME

Neurasthenia associated with what culture-bound syndrome:

SHENJING SHUAIRUO

In the US, what is the most common method of completed suicide?

SHOOTING ONESELF

The WHO has prioritized

to improve mental health services across the lifespan in low and middle income countries TRAINING NON-SPECIALIST PROVIDERS TO IDENTIFY AND TREAT MENTAL ILLNESSES

At the beginning of a radio talk show a psychiatrist states he is not entering into a doctor-patient relationship with any of the callers. Hereby he does not have

towards a patient in case of a malpractice suit (2x) DUTY

Initial step when child is referred for evaluation

CONSENT FOR EVALUATION FROM GUARDIAN

8 yo boy sees 2 bottles with same amount of liquid. The content of one is poured in shorter wider glass and the other to a longer narrow glass. When asked which has more liquid he says it's the same. According to Piaget he is exhibiting (2x)

CONSERVATION

What is achieved in Piaget's stage of concrete operations?

CONSERVATION

"4 yo child upset when ice cream melts. 10 yo puts it in the freezer and tells him it will be ok. ϰ yo insists it is ruined. Piaget's concept of this is:"

CONSERVATION (ALSO REVERSIBILITY)

What describes the implicit memory system?

CONSISTS OF MENTAL MODELS THAT SUMMATE MULTIPLE EXPERIENCES

Which validity assumes theoretical model of condition being measured and explanation of how components are related?

CONSTRUCT VALIDITY

What is the appropriate reaction to an HIV patient refusing to disclose their diagnosis to a sexual partner? (2x)

CONSULT STATE LAWS WHICH ADDRESS THIS ISSUE

Documents that first promoted first human subject consent in research

NUREMBERG CODE

Case Management/ C & L

Next Topic

Shrill noise reminds soldier of missile. What is the role of missile in classical conditioning?

UNCONDITIONED STIMULUS

According to psychoanalytic theory, the term primary process thinking refers to mental activity exemplified by (2x)

UNCONSCIOUS THOUGHTS THAT DO NOT MAINTAIN LOGICAL CONNECTIONS

According to psychoanalytic theory, the term primary process thinking refers to mental activity exemplified by which of the following?

UNCONSCIOUS THOUGHTS THAT DO NOT MAINTAIN LOGICAL CONNECTIONS

Psych MD is treating a child whose parents are divorced. Under what circumstance is it ethically acceptable for the psychiatrist to have a romantic relationship with the child's parent?

UNDER NO CIRCUMSTANCES

Per Piaget, conservation is the ability to do what?

UNDERSTAND THAT OBJECTS OR QUANTITIES REMAIN THE SAME DESPITE A CHANGE IN PHYSICAL APPEARANCE

Capacity eval component for referral to mental health court

UNDERSTAND THAT THE CHOICE OF THE OPTION INVOLVES WAIVING CERTAIN CONSTITUTIONAL RIGHTS

Hand washing rituals are most related to what defense mechanism? (4x)

UNDOING

In couples therapy, husband complains that MD wife works long hours, she says she stays late to make sure everything is "right" checking and rechecking her work. He notes that once in high school she wished her straight-A brother dead and later he died on a hunting trip. Her checking behavior is an example of what (2x)

UNDOING

"Famous entertainer comments publicly about having problems cutting down on ETOH use despite knowing negative effects on family members. Well-known psychiatrist, never met the entertainer, goes on a news program and says that the entertainer is most likely experiencing ETOH abuse. What explains the ethics of the MD? (2x)"

UNETHICAL BECAUSE THE PSYCHIATRIST HAS NOT EXAMINED THE ENTERTAINER

According to the World Health Organization, what is the number one psychiatric cause of loss of years of healthy life as measured by disability-adjusted life years for individuals between ages of 15 and 40 years?

UNIPOLAR MAJOR DEPRESSION

According to Kohlberg, the highest level of moral reasoning is based on consideration of

UNIVERSAL ETHICAL PRINCIPLES

What is most associated with high risk behaviors in adolescence?

UPBRINGING IN DISADVANTAGED NEIGHBORHOODS

78 yo w/ deteriorating personal hygiene, significant weight loss. Signs voluntary admission form but later forgets doing this. Potentially resectable masses in the lung and brain requiring further consent for surgical and oncological treatment. Consents but cannot explain the reason for surgery. Legally, psychiatrist should:

URGENTLY REQUEST PROBATE COURT APPOINTED GUARDIAN WITH POWER TO MAKE TREATMENT DECISIONS

27 yo angry, agitated pt is admitted to the ER. He is evaluated during the initial interview as non-psychotic and high risk for imminent violence. He refuses medication. What would be the most appropriate intervention at this time?

USE EXTERNAL RESTRAINT WITH CALM REASSURANCE

"Way to consider ͞learning styles͟ in which developing curriculum is best supported by literature"

USE SUPPLEMENTAL MATERIAL DESIGNED TO ADDRESS THE NEEDS OF ALL TYPES OF LEARNERS

"In forensic psychiatry, the term "standard of care" refers to the use of treatments that are (2x)"

USED BY AVERAGE REASONABLE PRACTITIONERS

Patient's confidentiality after death

USUALLY MUST BE MAINTAINED

According to classical psychoanalytic theory, what factor primarily accounts for the polarization of same sex peer groups?

UTILIZATION OF DEFENSE MECHANISMS TO AVOID SEXUAL IMPULSES

A psychiatrist and spouse are attending a concert. During intermission, the psychiatrist sees that they will cross paths in the lobby with one of the psychiatrist's patients. Which of the following would be the most appropriate conduct for the psychiatrist in this situation?

WALK BY WITHOUT INDICATING KNOWING THE PATIENT, UNLESS THE PATIENT INITIATES CONTACT

Gender is best understood as referring to

WAYS IN WHICH CULTURES DIFFERENTIATE ROLES BASED ON SEX

Example of type II error

FINDING ANTIDEPRESSANT INEFFECTIVE WHEN IT IS ACTUALLY EFFECTIVE

Defense mechanism when med student blames bad performance on attending not liking him?

RATIONALIZATION

Major psychological defense mechanism that determines the form and quality of OC symptom is

REACTION FORMATION

͞Substituted judgment͟ standard is based on this

WHAT THE DECISION MAKER BELIEVES THE PT WOULD PREFER

A finding regarding epidemiology of nicotine use:

YOUNG ADULTS REPORT THE HIGHEST RATES OF SMOKING

Appropriate response to immigrant patient with vague complaint

"""HOW WOULD YOU DESCRIBE YOUR PROBLEM TO A FAMILY MEMBER OR FRIEND?͟"

Which of the following patients would automatically be considered as lacking the capacity to give informed consent for medical procedures?

"A PATIENT WITH ALZHEIMER'S DEMENTIA WHO HAS A STATE DESIGNATED GUARDIAN"

Psychological function of a medication

"ACTS AS A CONTAINER FOR THE PATIENT'S PROJECTED ANXIETIESABOUT BEING DEFECTIVE"

"Psych MD sees 62 yo pt who has been hospitalized after an acute MI because staff report that ͞the pt is driving us crazy.͟ Nurses complain that pt is constantly requesting info about his condition and treatment, and will refuse tests and medications if he feels the explanations are not sufficient. On interview pt admits, ͞I am a perfectionist͟ but is proud of this fact, saying that it is ͞the secret of my success.͟ Pt is aware of the staff's frustration, but feels his requests for information are reasonable. The best advice to the team?"

"ANSWER THE PT'S QUESTIONS AND ACTIVELY INVOLVE THE PT IN ALL PORTIONS OF HIS TREATMENT"

75 yo pt with AD started on quetiapine 12.5 mg bid for agitation. According to Omnibus Budget Reconciliation Act, the MD must (2x)

"ATTEMPT TO DECREASE OR STOP THE MED DOSE AT LEAST EVERY 3 MONTHS."

During an evaluation for competency to stand trial, a defendant tells the court- appointed forensic psychiatrist about being involved in an unsolved crime. The psychiatrist should do what?

"AVOID A DETAILED DESCRIPTION OF THE DEFENDANT'S PAST OFFENSE HISTORY"

"With respect to Rotter's social learning theory, which of the following statements best describes having an internal locus of control?"

"BEHAVIOR IS SHAPED BY AN INDIVIDUAL'S EXPECTATION THAT IT WILL REALIZE A VALUED GOAL"

According to Winnicott, which is an essential component of parenting infants?

"BEING SENSITIVE TO THE CHILD'S INSTINCT FOR EXPLORATION AND GROWTH"

Investigators wish to maximize the probability of correctly rejecting the null hypothesis. This probability is referred to as:

1 MINUS BETA

Lifetime prevalence of schizophrenia

1%

"Compulsions & obsessions are related to development disturbance during which of Erikson's psychosocial stages?"

AUTONOMY VS SHAME AND DOUBT

Minimum age to meet Appelbaum criteria for decision making capacity is approximately

12 YEARS

Lifetime prevalence rate for ETOH abuse and dependence

15%

What is the prevalence of Alzheimer's in those х ϴϱ yo?

16-25%

"Calculate number needed to treat with lithium to prevent 1 suicide: 1000 subjects recruited in study, 500 receive placebo, 500 randomized to lithium. After 1 year, 50 from placebo commit suicide, 25 from lithium group suicide."

20

Which is a characteristic of assertive community treatment (ACT) to treat the severely mentally ill?

24-HOUR-A-DAY MENTAL HEALTHCARE COVERAGE

Achieving sense of self-control and free will, struggling between cooperation and willfulness (Erikson)

AUTONOMY VS SHAME AND DOUBT

Eriksonian phase corresponding with Freud's anal phase

AUTONOMY VS SHAME AND DOUBT

In an RCT which compares metformin addition to usual treatment in diabetes prevention, when 18% of patients in the metformin group develop diabetes vs 20% of patients in the usual treatment group develop diabetes, what is the NNT?

50

Twin studies of patients with schizophrenia have consistently identified substantial proportions of monozygotic twin pairs that are discordant for the illness. The most likely explanation is: (2x)

A "TWO-HIT" MODEL OF PATHOGENESIS EXISTS IN WHICH PREDISPOSING ENVIRONMENTAL FACTORS COMBINE WITH GENETIC RISKS

Assisted outpatient treatment (AOT) is what

A CIVIL COMMITMENT VIA COURT ORDER

16 yo caught for shoplifting jeans. No hx of stealing, jeans not her size. Pt frightened, remorseful, insomnia, failing grades, avoiding friends.

A CRY FOR HELP

In order to be judged competent to stand trial, a defendant must be able to consult with the lawyer with a reasonable degree of rational understanding and possess what?

A FACTUAL AND RATIONAL UNDERSTANDING OF PROCEEDINGS AGAINST HIM/HER

"Which of the following describes core narcissistic character pathology according to Kernberg's model?"

A FUSION OF THE IDEAL SELF, THE IDEAL OBJECT, AND THE REAL SELF RESULTING IN THE DEVALUATION OF OTHERS

What most seriously threatens the external validity of a research study? (3x)

A STUDY POPULATION THAT IS NOT REPRESENTATIVE OF THE POPULATION TO BE TREATED

What documentation will legally protect a psychiatrist, if pt dies by suicide?

A SUICIDE RISK ASSESSMENT AND PROTECTIVE FACTORS

Forcing med on pt directly challenges which of the following?

AUTONOMY

What risk management strategy for dealing with a pt with SI is most likely to be rejected as invalid in a subsequent malpractice lawsuit?

A ͞NO-HARM͟ OR SUICIDE PREVENTION CONTRACT BETWEEN THE PT AND THE THERAPIST

Providing pts with an understanding of their disorders and options for treatment is honoring which of the following principles of ethics

AUTONOMY

Which ethical principle supports a parents keeping a secured firearm in the house despite having a suicidal child?

AUTONOMY

In a patient with preexisting depression to a terminal illness, requesting euthenasia would bring what two ethical principles into opposition?

AUTONOMY AND NONMALEFICENCE

Cognitive therapy was developed by who (2x)

AARON BECK

Pt attempted suicide during a crisis which was worsened by pt's feelings about being unable to reach vacationing psychiatrist who did not provide coverage. What is the psychiatrist's vulnerability?

ABANDONMENT

According to the theory of self-psychology, a major cause of mental illness is

ABSENCE OF AGE-SPECIFIC MIRRORING RESPONSES

"What is the proper response to an offered low monetary gift (cookie) from a schizotypal patient?"

ACCEPT COOKIE AND THANK THE PATIENT

Example of boundary crossing by a psychiatrist rather than a boundary violation:

ACCEPTING A COOKIE FROM A GUARDED, PARANOID PATIENT

504 plans ensure what for students with disabilities?

ACCOMMODATIONS NECESSARY TO MEET EDUCATIONAL NEEDS

30 yo MD who has emigrated from China to the US continues to take pride in family recipes, but has recently embraced American ballroom dancing. This represents what?

ACCULTURATION

"Which is listed on the President's New Freedom Commission on Mental Health (2003)? (2x)"

ADDRESSING MENTAL HEALTH AS VIGOROUSLY AS PHYSICAL HEALTH

In psych testing, how is content validity shown?

ADEQUATE SAMPLE OF AREA OF INTEREST

SI pt needs hospitalization but insurance thinks partial. What to do?

ADMIT PT TO HOSPITAL AND APPEAL INSURANCE

Why don't twin adoption studies have methodological limitations?

ADOPTEES ARE NOT A REPRESENTATIVE POPULATION SAMPLE

Adoption research in one method to delineate genetic vs environmental influences on phenotype. Limitation of this approach?

ADOPTEES ARE NOT A REPRESENTATIVE SAMPLE AND ADOPTIVE AND BIOLOGICAL PARENTS MAY RESEMBLE ONE ANOTHER

What research design best clarifies the role of environment in behavioral genetics?

ADOPTION STUDY

"What can be used when a person wishes to assign a surrogate to make healthcare decisions during future periods of mental incapacity?"

ADVANCE DIRECTIVE

"Which legal mechanism best conveys a dying patient's instructions regarding medical intervention when unable to make decisions?"

ADVANCE DIRECTIVES

What response would be based on a purely liaison model of psychosomatic medicine for a pt who may be depressed?

ADVISE THE TEAM ON HOW TO PROPERLY DIAGNOSE DEPRESSION

Psych MD is approached at a social gathering by a neighbor who seeks advice about recent Sx of depression and anxiety that neighbor has experienced since being fired from work. Which action would constitute establishment of a doctor-pt relationship?

ADVISING NEIGHBOR TO START CBT WITH ONE OF SEVERAL PROVIDERS

Gag-rule clauses in some managed care plans may prohibit docs enrolled in their plans from taking which action?

ADVISING PTS ABOUT TREATMENTS NOT COVERED BY HMO

Biological consequences of psychological stress are documented to affect what

ALTERATION OF NEURO-IMMUNO-ENDOCRINE FUNCTION

According to sociobiologic theory, what term describes behaviors at the level of the individual that maximize fitness at the level of the gene?

ALTRUISM

Company X develops new medication for depression. Although the initial trials reveal only modest, statistically insignificant mood benefits, the research team decides to analyze the drug's effects in subgroups. By subgrouping the subject pool into 20 subgroups, the researchers find statistically significant mood benefits for men/women w/ birthdays in the early fall. Conclusion that can be drawn from finding is that it is:

AN ARTIFACT OF MULTIPLE ANALYSES

"Medication can be administered without a patient's consent under which of the following circumstances? (2x)"

AN EMERGENCY SITUATION EXISTS AND CAN BE DOCUMENTED

To whom is specific consent needed to share individually identifiable health information under HIPAA

AN INVESTIGATOR FOR THE PURPOSE OF CONDUCTING EPIDEMIOLOGICAL RESEARCH

Classical psychoanalytic theory says pleasure an adult might take in controlling others/making order of chaos relates to which psychosexual stage of development?

ANAL RETENTIVE

Groups of pts w/ MDD, dysthymic d/o, and adjustment d/o were given mood scales. In order to compare the means of scores of these three groups, most appropriate statistical methods is what (3x)

ANALYSIS OF VARIANCE

"3 yo girl hurts self with a tricycle. Then she hit the tricycle and asks, ͞Why did you hurt me?͟ Which thought process does this behavior exemplify?"

ANIMISM

Example of mature defense mechanism in classical psychoanalytic theory:

ANTICIPATION

Psychiatric disorder with highest prevalence

ANXIETY DISORDERS

In a properly conducted randomized trial, why don't you include P- values in the baseline characteristics table?

ANY DIFFERENCES AT BASELINE MUST HAVE ARISEN BY CHANCE GIVEN RANDOMIZATION

In ethics, beneficence is: (6x)

APPLYING ONE'S ABILITIES SOLELY FOR THE PATIENT'S WELL-BEING; PHYSICIAN ACTS PATERNALISTIC

The criminal defendant tells the psychiatrist doing a forensic eval ͞I know that killing my father was illegal.͟ However, the defendant delusionally believed that the father was being tortured by demons and killing him was the moral thing to do. The defendant is unable to do what?

APPRECIATE THE WRONGFULNESS OF HIS CONDUCT

Which ethnic group in the US has the lowest 12 month prevalence of alcohol use disorder (2x)

ASIAN AMERICANS AND PACIFIC ISLANDERS

"20 yo western trained psychiatrist of northern European descent has been asked to treat a 40 yo Hindu patient who emigrated to the US 2 years ago. The patient presents with complaints of weakness, fatigue, headaches, insomnia, palpitations, and decreased appetite. The patient also reports a 5 lb weight loss over the last month, dysphoric mood with intermittent anxiety, and suicidal ideation. In addition to assessing the patient's level of suicidality, it would be appropriate for the psychiatrist to do which of the following when beginning treatment? (3x)"

ASK THE PATIENT WHAT HE BELIEVES EXPLAINS THE SYMPTOMS

Most effective in reducing inpatient hospital use, increasing housing stability, controlling psychiatric sx, and improving quality of life for the severely mentally ill?

ASSERTIVE COMMUNITY CARE

29 yo M pt with hx of CPS 8x hospitalization in 14 months, stopped his meds and is unable to take care of himself, becomes paranoid and stays on the street, eating out of garbage cans and his family is unable to support him anymore. Most appropriate intervention to decrease this pt's risk of future hospitalizations ;ϮxͿ

ASSERTIVE COMMUNITY TREATMENT

Involuntary outpatient commitment is most effective when

ASSERTIVE COMMUNITY TREATMENT IS INCLUDED

Dementia patient falls and refuses recommended intervention. First step? (2x)

ASSESS PATIENT'S CAPACITY TO REFUSE

Dementia pt with a durable power of attorney, decisions need to be made by?

ASSIGNED DECISION MAKER

If pt is incompetent to make decision and has durable power of attorney, treatment recommendations will have to approved by

ASSIGNED DECISION MAKER

Researchers trying to determine frequency of disease within population thought to have commonly inherited variants of relatively small effect. Most appropriate study?

ASSOCIATION ANALYSIS STUDY

Regarding informed consent ͞therapeutic misconception͟ means

ASSUMPTION BY RESEARCH PARTICIPANTS THAT DECISIONS ABOUT THEIR CARE ARE BEING MADE SOLELY WITH THEIR BENEFITS IN MIND

The most appropriate time to discuss advance directive

AT THE TIME OF ADMISSION TO THE HOSPITAL

Diagnosis of culture-bound syndrome for a pt with insomnia, HAs, anorexia, fears, despair, diarrhea, & anger?

ATAQUE DE NERVIOS

Ainsworth's strange situation experiment evaluated what?

ATTACHMENT BETWEEN PARENT AND CHILD

Difference between incidence rates

ATTRIBUTABLE RISKS

27 yo schiozphrenic pregnant women at 23 weeks in ED reports cramping, refuses pelvic exam, ED doc says decisional capacity intact. Which ethical principle is being applied to her decision?

AUTONOMY

Adolescent with recent SA is about to be discharged from hospital. The parents refuse a request to remove firearms from the house, stating, ͞I need firearms to protect my family and I will make sure they are secured.͟ Which ethical principle CONFLICTS with the psychiatrist making a CPS report?

AUTONOMY

Failing to obtain informed consent defies which principle of ethics?

AUTONOMY

Method used to reduce false positives with a smaller P value

BONFERRONI CORRECTION

"Researchers wish to study a group of students who were exposed to a terrorist attack in which some of their classmates were killed and compare the students to other students who had not undergone such a trauma. For the study to be considered quasi-experimental, which of the following requirements must be met?"

BASELINE DATA MUST HAVE BEEN COLLECTED BEFORE THE EVENT

If interviewing an agitated patient

BE AWARE OF BODY POSITION OF SELF AND PT

Pts who are committed to psychiatric treatment institutions in most states are presumed to

BE COMPETENT TO MAKE THEIR OWN DECISIONS

A child with autism is placed in a therapeutic foster home where a consistent response is made to a given action with the goal of improving the child's functioning. This technique derives from which of the following learning theories? (2x)

BEHAVIORISM

Psychic determinism is (2x)

BEHAVIORS RESULT FROM UNCONSCIOUS MIX OF DRIVES, DEFENSES, OBJECT RELATIONSHIPS, SELF-DISTURBANCES

What would be a situation in which to report a colleague's behavior?

BEING AGGRESSIVE TOWARD PATIENTS ON THREE DIFFERENT OCCASIONS DURING SHIFTS IN THE ED

Pt who suffers from a serious mental d/o is a defendant in a criminal matter. Even if pt understands the nature and objectives of court proceedings, pt may be found not competent to stand trial under

BEING UNABLE TO ASSIST ATTORNEY IN THE DEFENSES

Individual with schizophrenia arrested after breaking into grocery store would be found incompetent to stand trial if s/he

BELIEVES ALL LAWYERS ARE DEMONS AND REFUSES TO SPEAK TO THEM

Ethics of pursuing inpatient psych for SI pt refusing tx?

BENEFICENCE

Paternalism can be justified by which of the following basic ethical principles (2x)

BENEFICENCE

What ethical principle provides the most appropriate basis for psychiatric intervention in a mentally incompetent patient? (6x)

BENEFICENCE

According to contemporary psychoanalytic theory, from birth to 18 months, children experience an emerging ͞self͟ as a result of what event?

BIOLOGICAL PROCESSES AND BODILY SENSATIONS COME TO HAVE A PSYCHOLOGICAL MEANING

A significant risk factor independent of depression for postpartum depression is

C-SECTION DELIVERY

In a randomized controlled trial, one group got usual antipsychotic and benzos for rescue. The experimental group got new antipsychotic and NO rescue. Group responses equal. What threatens validity in this study?

CONFOUNDING VARIABLE

24 yo homeless with SI requests hospitalization. This is 3rd visit in the span of 2 weeks. Pt is organized, coherent. When asked what would be different with this hospitalization, attacks the doctor. Best next step of action?

CALL LAW ENFORCEMENT

HIPAA allows PCP and psychiatrist to discuss what

CAN DISCUSS THE CASES THAT PCP IS INVOLVED WITH

48 year old with metastatic breast cancer hospitalized for AMS, struggling to stay awake or attend to conversations. Medical team is concerned for brain mets. When asked to consent for MRI, the patient says ͞don't know, don't know.͟ Psychiatry does capacity assessment and determines that she is delirious and does not have capacity to make decisions. What should the psychiatrist tell the team?

CHECK TO SEE IF PATIENT HAS AN ADVANCE HEALTH CARE DIRECTIVE

Which is used to evaluate the relative frequencies or proportions of events in 2 populations that fall into well defined categories/two dichotomous variables

CHI - SQUARE

What situation requires the psychiatrist to step out of the usual treatment relationship to protect the pt or others from harm?

CHILD ABUSE

According to Freud, which of the following is considered to be the most salient feature of normal psychosexual development in children between 1-3 years of age?

CHILD-PARENT STRUGGLES ABOUT THE NEED TO DELAY GRATIFICATION OF DESIRES

Learned helplessness is based on principles of

CLASSICAL CONDITIONING

What is the legal term for the minimal degree / level of certainty needed for placement of an involuntary hold?

CLEAR AND CONVINCING EVIDENCE

Clinical studies have found out what about telepsychiatry when used for psychiatric patients

CLINICAL ASSESSMENTS ARE AS ACCURATE AS IN-PERSON ASSESSMENTS

Pt with a history of bipolar disorder hospitalized 2 weeks ago for mania and SI. Improved but 2 days before being allowed home visit, patient stops all meds. Doctor states that if patient refuses meds, cannot go home for visit. Patient relents and accepts meds after hearing this. From ethical standpoint, response from doctor can be considered as what

COERCIVE BUT ACCEPTABLE IF THE DOCTOR BELIEVED NON- ADHERENCE WOULD CAUSE SIGNIFICANT HARM

According to Aaron Beck, the primary defect in depression involves what

COGNITIVE DISTORTION

"A group of researchers studied the temperament of a group of children at age 3 months, 2 years, 5 years, and 20 years to determine relationships between initial temperamental characteristics of children who eventually had psychiatric problems. This is an example of what type of study? (2x)"

COHORT

Best study design to evaluate incidence of depression in those that lose job vs those with jobs

COHORT

Study design that answers whether exposure to a drug increases risk of an adverse side effect

COHORT

What research design would be the most appropriate in establishing a causal relationship between childhood vaccination and onset of autism?

COHORT

What research study examines a group studied over a prolonged time period? (6x)

COHORT

"Research suggests that the frequency of alcoholism in lesbians as compared to heterosexual women was greater only in lesbians over 55 yo. Which of the following is considered to be the best explanation for this finding?"

COHORT PHENOMENON RELATED TO SOCIOCULTURAL ISSUES

Type of study to determine relationship between risk factor and development of disease

COHORT STUDY

Existing mental health care programs that do not have the capacity to provide effective treatment to all patients in need would be a primary rationale for implementing what?

COLLABORATIVE CARE

What did the CATIE trial demonstrate about second generation antipsychotics for treatment of schizophrenia?

COMBINATIONS OF FIRST AND SECOND GENERATION ANTIPSYCHOTICS WERE VERY EFFECTIVE

How should a psychiatrist handle an interview about the misbehavior of a prominent government employee?

COMMENT ON HUMAN BEHAVIOR GENERALLY, BUT REFUSE TO OFFER OPINIONS ABOUT THE SPECIFIC PERSON

Informed consent is NOT required in what forensic evaluation

COMPETENCY TO STAND TRIAL

Does the law presume a person is competent or incompetent before other information is known?

COMPETENT

Which is part of ABPN board maintenance & certification program: Completing patient safety activity, Completing 15 CME, Treating minimum of 25 patients/year for 10yrs, Having active medical staff privileges (2x)

COMPLETING PATIENT SAFETY ACTIVITY

Most significant barrier to accessing mental health services in school- based health centers

CONCERNS ABOUT CONFIDENTIALITY

"In which stage of Piaget's theory of cognitive development do children first understand conservation of volume?"

CONCRETE OPERATIONAL

What is the combination of several unconscious impulses, wishes, or feelings that are attached to a single dream image?

CONDENSATION

Nausea to apple juice after vomiting from apple-flavored liquor in the past. What aspect of learned behavior?

CONDITIONED STIMULUS

Method used to compute probability estimates to allow researchers to generalize from sample population to larger population from which samples were drawn?"

CONFIDENCE INTERVALS

Psychiatrist calls pt at home; wife asks how pt is doing. What principle governs response?

CONFIDENTIALITY

The psych MD who released medical records and audiotapes of the treatment of the poet Anne Sexton to the author who was writing a biography after her death could have been criticized for violating what principle?

CONFIDENTIALITY

Experimental subjects were asked to make a judgment but gave a wrong answer in spite of knowing the right one because they didn't want to disagree with responses of other participants. This phenomenon is:

CONFORMITY

Study compares pregnant women taking antidepressants for depression with controls. Study reports association between antidepressant use in pregnancy and mental health diagnoses in offspring. Which statistical fallacy is this an example of?

CONFOUNDING

Recommended use of ͞copy forward͟ in a progress note?

DATA COPIED FORWARD SHOULD BE ESSENTIAL AND PERTINENT TO THE CURRENT ENCOUNTER

"Which research or statistical methodologies were used to develop the DSM-IV-TR?"

DATA RE-ANALYSES AND FIELD TRIALS

"14 yo Native American Indian girl brought to mental health center b/c she has visions of spirits. Tells psychiatrist spirits are elders who appear, give her advice. No hx of substance abuse/mental disorder. Interview w/ parents reveals they are traditional Θ feel daughter's visions are a gift. But they are worried about schoolwork, which has deteriorated b/c she has been preoccupied with spirits. Tx should focus on: (3x)"

DEALING WITH HER SCHOOL PROBLEMS

According to Holmes & Rahe, what life event is associated with highest stress/disruption

DEATH OF A SPOUSE

"What finding in the CATIE-AD trial was demonstrated in subjects using antipsychotic medications for their behavioral disturbances by week 12?"

DECREASED LEVEL OF SUSPICION

"In terms of the normal functioning of the personality, Anna Freud's contributions were in the realm of"

DEFENSE MECHANISMS

Most important finding for a psychiatrist to consider in disability determination

DEGREE OF FUNCTIONAL IMPAIRMENT

"7 yo states he knows dad died, but why didn't he come to the birthday party? Defense mechanism?"

DENIAL

Pt's wife states he drinks almost every night. Pt states he never drinks to excess, never drinks outside the home, never needs an eye opener, and drinking does not affect his work performance. This is an example of which defense mechanism?

DENIAL

Learned helplessness is a model for what disorder (2x)

DEPRESSION

According to WHO, the world's greatest cause of mortality, ill health and suffering is what?

DEPRESSION AND SUICIDE

In the global burden of disease study series, which disorders have the highest measurement of disability adjusted life years (DALYs)? (2x)

DEPRESSIVE DISORDERS

Residents are working on QI project to improve missed handoffs. Aware that this is a systemic problem. Next step?

DESIGN A PROTOCOL TO DETERMINE THE PREVALENCE OF MISSED HANDOFFS

First step when determining whether or not a criminal defendant is not guilty by reason of insanity (2x)

DETERMINE THE LEGAL INSANITY STANDARD TO BE USED

What documentation will legally protect a psychiatrist if pt dies by suicide? (2x)

DETERMINE THE LEGAL INSANITY STANDARD TO BE USED

Pt is annoyed by the family's expression of concern for his condition, saying, ͞what they are saying is all in the talk.͟ What defense mechanism is this?

DEVALUING

Most accurately describes the primary function of a mental health court

DIRECTS MENTALLY ILL DEFENDANTS TO TREATMENT RATHER THAN PUNISHMENT

"Psych eval requested on management of 32 yo pt admitted by internal medicine for tx of severe gastrointestinal viral infectionͶ dehydration, nausea, vomiting, fever. Pt has hx of schizoaffective disorder, controlled w stable doses of clozapine over the past year. Staff concerned that pt might be delusional/hallucinating, as pt has appeared somewhat confused/inattentive. On exam, pt is oriented to person, place but not time. Pt somewhat slow, appears visibly ill/tired. Pt complains of stiffness, there is some rigidity to movements. Psych recommendation?"

DISCONTINUE CLOZAPINE

Past years show that annual prevalence was much higher than annual incidence

DISEASE IS CHRONIC

Sports fan furious with a spectator for interfering with a play that could have won the game. Denigrates the spectator on social media. Defense mechanism?

DISPLACEMENT

Whenever the culture of western medicine has been a focus of inquiry by anthropologists, what diagnosis has been seen as a culture bound syndrome in North America?

DISSOCIATIVE IDENTITY DISORDER

Resident is considering using an Internet search to gather information about a patient. In which of the following situations is this most appropriate?

DISTINGUISHING BETWEEN THE PATIENTS GRANDIOSE DELUSIONS AND REALITY

Primary purpose of mental health courts

DIVERT OFFENDERS WITH MENTAL ILLNESS AWAY FROM INCARCERATION

Recommendation in the guidelines published by the AMA in regard to social media use?

DOCUMENTATION OF ALL ELECTRONIC PATIENT COMMUNICATION IN THE MEDICAL RECORD

Patient refusing IV antibiotics, what is a reason he would not have capacity to do this?

DOES NOT UNDERSTAND CONSEQUENCES OF REFUSING TREATMENT

A crossover design is a variation of what

DOUBLE-BLIND

Type of study design most likely to eliminate the bias of knowing what treatment is given to the pt

DOUBLE-BLIND

Freud's psychological theory of development ;ϮxͿ

DRIVE THEORY

Clinical trial: 2 study meds in 8 hospitals. 20 receive drug 1 in hospitals A,B,C,D, while 20 receive drug 2 in hospitals E,F,G,H. Drug 1 is proven and indicated treatment. Which critical confounder biases the results?

DRUG ASSIGNMENT DEPENDS ON PARTICIPANT'S HOSPITAL

Pt arrested for possession of cocaine, spends 2 days in jail, after missing appointments with substance abuse counselor. Intervention is referred to as what

DRUG COURT

Multiaxial system was introduced in what edition of DSM

DSM III

First edition of the DSM to exclude homosexuality as a diagnosis

DSM-III-R (1987)

Immigrants to the US experience stress & demoralization most during which period?

DURING FIRST 1-3 YEARS IN THE COUNTRY

"A patient requests an alternative tx for depression. His psychiatrist does not practice this tx himself but refers the pt to one who does. In 6 months the pt has complications from the tx and sues the 1st psychiatrist for malpractice. Which element of malpractice is missing in this case?

DUTY

Melanie Klein differed from Freud in her emphasis on what factor?

EARLY OBJECT RELATIONS

Which statistic measures the magnitude of difference between two intervention groups? (3x)

EFFECT SIZE

"Naturalistic investigations of a treatment's effects look at which factor?"

EFFECTIVENESS

"Researcher wants to compare a new atypical antipsychotic to the already existing atypical antipsychotics. Researcher wants to conduct the study in routine clinical situations rather than using highly selected subjects. Which of the following study designs would be the most appropriate? (2x)"

EFFECTIVENESS

Recently a retired woman with recent life transitions has regrets about job and daughter. Wants meaningful last few decades. What developmental stage?

EGO INTEGRITY VS DESPAIR

What psychotherapeutic school of thought emphasizes how a pt has managed feelings of envy/rage with particular emphasis on defense mechanisms?

EGO PSYCHOLOGY

There are ϯ primary components of ͞meaningful use͟ of electronic medical records (EMR). These include certified EMR in a meaningful manner, use of certified EMR technology to submit clinical quality measures, and what?

ELECTRONIC EXCHANGE OF HEALTH INFORMATION TO IMPROVE QUALITY OF CARE

The purpose of designing a study to use the double blind method is to (2x):

ELIMINATE BIAS DUE TO EXAMINER EXPECTATIONS

Therapy intervention most frequently associated with Kohut's self- psychology

EMPATHIC VALIDATION

"What is the strategy of the ͞Health Homes͟ established by the Affordable Care Act of 2010?"

EMPLOYING A WHOLE-PERSON APPROACH WITH CONNECTIONS TO COMMUNITY CARE, AND SOCIAL AND FAMILY SERVICES

Core characteristic of the "system of care" movement in community mental health?

EMPOWERING FAMILIES TO PARTICIPATE IN TREATMENT PLAN DECISIONS

According to Winnicott's theory of the development of an infant's assertiveness and sense of competence, what best describes the critical function of the holding environment?

ENCOURAGES SPONTANEITY WHILE PROTECTING THE CHILD FROM HARM

HIPAA regulations guiding use of email in patient-doctor communications

ENCRYPTION IS NOT REQUIRED BUT AMOUNT AND TYPE OF INFORMATION DISCLOSED IN EMAIL SHOULD BE LIMITED

Mental health courts increase what outcome:

ENGAGEMENT IN COMMUNITY TREATMENT

11 yo boy has frequent episodes of ulcerative colitis requiring frequent hospitalization. While in the hospital, mother never leaves his side and responds to questions for him, often referring to disease as "our disease." According to Minuchin's theory of family interactions, this is: (3x)

ENMESHED

Which entity mandates public schools to provide educational services to children with disabilities?

FEDERAL GOVERNMENT

Erikson theorized that a successful developmental task in the 40-60 yo group is to? (4x)

FEEL USEFUL TO SOCIETY THROUGH BEHAVIORS THAT PROTECT FUTURE GENERATIONS

Pt suffers as a result of the pt's psychiatrist going on vacation wͬo providing coverage. The psychiatrist's problematic behavior violates which of the following ethical principles?

FIDELITY

A psych consultation is requested for a patient whom the medical team believes is manufacturing symptoms of an illness. Which of the following factors would be most consistent with a diagnosis of malingering?

FINANCIAL INCENTIVE FOR ILLNESS

What best characterizes the current explanation for group differences in prevalence of psychiatric illness?

FACTORS THAT PROMOTE VULNERABILITY TO STRESS

Best indication for residential treatment in substance abuse? (3x)

FAILURE TO MAINTAIN ABSTINENCE AFTER TREATMENT IN INPATIENT SETTINGS

Sufficient to establish that a person is incompetent to stand trial (3x)

FAILURE TO UNDERSTAND THE CHARGES ONE IS FACING

"According to Winnicott, if mother can't provide ͞holding͟ environment for infant, infant will develop what defensive process?"

FALSE SELF

"Assessment instrument used in epidemiology study that relies solely on subjects' report"

GENERAL HEALTH QUESTIONNAIRE

"Erickson's psychosocial stage in which a person invests energy into establishing, caring for, and guiding in the next generation? (2x)"

GENERATIVITY VS STAGNATION

HIPAA requires physicians to do what regarding privacy rights for patients:

GIVE WRITTEN NOTICE OF PATIENT'S PRIVACY RIGHTS

5 yo boy becomes restless, impulsive, and difficult to manage when not given sufficient exercise. Teacher gives him increased motor activity, and his behavior improves. This is an example of: (2x)

GOODNESS OF FIT

How is ethnicity defined? (4x)

GROUP OF INDIVIDUALS SHARING SENSE OF COMMON IDENTITY, COMMON ANCESTRY, SHARED BELIEFS, & HISTORY

Tendency for groups to arrive at more extreme decisions than for individual group members alone

GROUP POLARIZATION

An elderly patient with advanced dementia refuses nursing home placement. Recommended guardianship?

GUARDIAN OF PERSON

Name the agency that inspects hospitals, sets standards of care and performance to prevent medical errors?

JOINT COMMISSION

Mental competency of an elderly patient is determined by (8x)

JUDICIAL HEARING

Research in ADHD kids to argue medications should be available to all children to improve function rests on which ethical principle?

JUSTICE

The efforts of psychiatrists to achieve ͞parity͟ legislation for mental illness can best be seen as illustrating which of the following moral principles?

JUSTICE

A psychiatrist employed by an institution wants to use pt data for research later. How can she get the progress notes?

KEEP HER OWN SEPARATE RECORDS

Acculturation outcome associated with high risk for suicide, substance abuse, and alcoholism (2x)

LACKING STRONG ALLEGIANCE TO NATIVE CULTURE, AND NOT INCORPORATING HOST CULTURE

Which statistical method attempts to address the effects of participants dropping out of a study prior to completion? (7x)

LAST OBSERVATION CARRIED FORWARD

Developmental stage that panic d/o presents?

LATE ADOLESCENCE

Model to identify problem areas and workflow and eliminating potential waste (i.e. non-value added activities)? (2x)

LEAN METHODOLOGY

Pt's family believes depression is due to santeria curse. During assessment, psych should focus on what

LEARNING FROM PT AND FAMILY WHAT THEY BELIEVE THE COURSE OF THIS ILLNESS IS EXPECTED TO BE

Main advantage of family-based genetic association studies over case- control association studies

LESS AFFECTED BY SYSTEMATIC POPULATION DIFFERENCES

What reduces risk of liability in collaborative care?

LIMIT PRACTICE TO FORMAL CONSULTATIONS AND REFRAIN FROM CURBSIDE CARE

Pt is admitted to ICU. Pt can communicate but is deemed incompetent. Which of the following advance directives should be used to inform the hospital staff of pt wishes?

LIVING WILL

Female who drinks with fhx of etoh-associated cancer, wants to know if abstinence decreases risk of CA

LONG TERM ABSTINENCE WOULD REDUCE RISK SIGNIFICANTLY

Intimate partner violence is most common in which demographic

LOW INCOME FAMILIES

Per WHO, most common barrier to getting mental health treatment

LOW PERCEIVED NEED

Demographic feature associated with persistence of symptoms in adults with somatic symptom disorder?

LOWER EDUCATIONAL LEVEL

Prevalence of psychotic disorders after age 65 is

LOWER IN BOTH MEN & WOMEN

"When compared to younger age groups, the prevalence estimates of MDD in persons older than 65 years are"

LOWER IN BOTH MEN AND WOMAN

"29 yo severely depressed with SI. Threatens to kill his wife and daughter to ͞take them with me.͟ Legal responsibility is what"

NOTIFY THE WIFE

Assuming there is no significant difference between 2 random samples of population is:

NULL HYPOTHESIS

According to Kohut, which trait is normal in children and is a significant aspect of healthy personality development?

NARCISSISM

What is the most prominent lobbying and support organization for families of people with severe psychiatric disorders?

NATIONAL ALLIANCE FOR THE MENTALLY ILL

Females of which ethnicity report highest rates of intimate partner violence?

NATIVE AMERICAN

Racial/ethnic group with highest rate of suicide? (3x)

NATIVE AMERICANS

Research into the complexity of healthcare systems and distribution of resources is best carried out in which fashion?

NATURALISTIC STUDY

80 yo pt presents with sudden onset seizures, hallucinations and aggressive behavior. Preliminary CT scan of the head does not show abnormalities. The pt's family wants to know what is the likelihood that the pt has no changes in his brain given the CT scan. Which of the "following concepts would need to be used to respond to the family's concerns about the CT scan results?"

NEGATIVE PREDICTIVE VALUE

40 yo with recurrent panic attacks in public places, what type of learning pattern is demonstrated when they leave areas where the panic attack begins then the panic attack stops?

NEGATIVE REINFORCEMENT

Patient presents for initial evaluation of OCD, started after sibling hospitalized for severe infection and now they are worried about contracting an infection and compulsively washes hands to eliminate the fear. From learning theory perspective what does the hand washing represent?

NEGATIVE REINFORCEMENT

Most common factor in cases of termination of parental right (2x)

NEGLECT BY THE PARENT

Term when a physician fails to exercise the standard of care

NEGLIGENCE

Puerto Rican and other Latin American pts are more likely than those from western culture to report their depression as what? (2x)

NERVOUSNESS

Disassociation is what type of defense mechanism?

NEUROTIC DEFENSE

Which are the dimensions of the Five Factor Model of Personality?

NEUROTICISM, EXTRAVERSION, OPENNESS, AGREEABLENESS, CONSCIENTIOUSNESS

How to manage phone conversation with a labile patient who ended a relationship with boyfriend and is having SI with plan, but is willing to stay with family and allows you to speak with family?

NEXT DAY OUTPATIENT FOLLOW UP WITH PSYCHIATRIST

Fact that participants did NOT receive available indicated treatment in the Tuskegee Syphilis study is an ethical violation of justice, beneficence or non-maleficence?

NON-MALEFICENCE

Patient threatens criminal behavior toward property of another. Legal obligation of psychiatrist?

NONE

"First do no harm" refers to what ethics principle ;ϰxͿ

NONMALEFICENCE

A patient has refractory depression. The psychiatrist isn't sure what to try next and consults a supervisor. What ethical principle is illustrated by the consult?

NONMALEFICENCE

Watson's experiment with "Little Albert" demonstrated that an ϭϭ yo child, who previously showed no phobic response to a stimulus, learned to fear the stimulus after it was repeatedly paired with a loud noise. This would be unethical b/o violation of: (2x)

NONMALEFICENCE

In the analysis of results of the national comorbidity study (1994), a nonresponse adjustment weight was included. This inclusion was important for what reason?

NONRESPONDERS WERE FOUND TO HAVE HIGHER RATES OF ILLNESS

Research shows dissociative symptoms most commonly represent what worldwide?

NORMAL PART OF RELIGIOUS/RITUAL EVENT

"APA addressed ethics of sexual relationships between psychiatrists & pts by stating"

NOT PERMISSIBLE TO HAVE A SEXUAL RELATIONSHIP WITH A CURRENT OR FORMER PATIENT

What protects a psychiatrist from not reporting a regretful mother accompanied by her visibly injured child hurt from child abuse?

NOTHING, LEGAL ACTIONS MAY FOLLOW

Early behaviorist theory promoted what

OBJECTIVE PSYCHOLOGICAL RESEARCH

"Pt called ED complaining of hallucinations that command the pt to kill others. Psychiatrist's first action?"

OBTAIN PHONE NUMBER, ADDRESS OF PATIENT

In some Japanese and Korean cultures, rather than an intense fear of embarrassing oneself socially, social phobia symptoms may instead manifest with intense fear of what?

OFFENDING OTHERS

Licensing requirements in telepsychiatry

ONLY IN THE STATE WHERE PATIENT IS LOCATED

Most ethical action when psychiatrist receives subpoena for records pertaining to patient arrested for selling marijuana

OPPOSE SUBPOENA DUE TO LACK OF PT CONSENT

In Heinz Kohut's theories of self-psychology, what is meant by the term "self-objects"?

OTHER PEOPLE IN THE ENVIRONMENT WHO PERFORM PARTICULAR FUNCTIONS FOR THE SELF

Pt sadly reports receiving a college rejection letter, and then disparagingly predicts that all the other colleges applied to will reject him as well. This is an example of

OVERGENERALIZATION AND ALL-OR-NONE THINKING

48 yo with chronic sadness and no relationships since her husband died 12 years ago. Normal grieving, no DSM-IV criteria for d/o, has successful career and close friends. States having conversations with her dead husband and he talks to her. Believes in heaven. Likely classification of these experiences is:

PART OF A BELIEF SYSTEM ENDORSED BY HER RELIGION

According to APA, it is unethical for a psychiatrist to

PARTICIPATE IN LEGALLY AUTHORIZED EXECUTION

In determining the disclosure of psychiatric treatment information in court, which individual is considered to hold privilege

PATIENT

Pt seen psychiatrist once, then psychiatrist only calls to follow-up, but doesn't take on pt care

PATIENT ABANDONMENT

Best defense against medical malpractice? (2x)

PATIENT DID NOT SUFFER A BAD OUTCOME

Psychiatrist thinks patient under managed care plan will need combined psychopharm and psychotherapy for > 1yr. Knows that the health plan stops paying after 20 visits. Most thing ethical to tell patient

PATIENT MAY NEED MORE TX THAN INSURANCE COVERS

A neighbor asked a psychiatrist that lives next to him about increasing his prozac to 20 mg because of increased depression. This establishes what between the neighbor and psychiatrist?

PATIENT-PHYSICIAN RELATIONSHIP

"HIPAA limits disclosure of patient health information without patient's consent except for what"

PAYMENT

What is an unethical fee arrangement for a forensic psychiatrist? (2x)

PERCENTAGE OF PLAINTIFFS AWARD

Legal expectation of psychiatrist in the evaluation of a patient with suicidality

PERFORM AND DOCUMENT COMPETENT SUICIDE RISK ASSESSMENT

According to Erikson, the predominant emotional issue in normal, school-aged children (3x)

PERSONAL WORTH AND COMPETENCE

Risk factor most predictive of a pt having emergent SI:

PERVASIVE INSOMNIA

Pt w/ severe GI disease requests a surgical procedure with questionable efficacy and a relatively high risk of negative effects. Level of competency a psychiatrist would require finding pt competent to make treatment decision?

RATIONAL CONSIDERATION OF ALTERNATIVES

A bookkeeper who is embezzling money justifies behavior by noting company is still making a profit. Defense mechanism?

RATIONALIZATION

Frequency with which interdisciplinary team should evaluate cognition and affective functioning of nursing home pts

QUARTERLY

Best reported outcome of mental health courts

QUICKER ACCESS TO COMMUNITY TREATMENT

Difference between ͞race͟ and ͞ethnicity͟

RACE IS BASED ON PHYSICAL CHARACTERISTICS WHEREAS ETHNICITY INCORPORATES MULTIPLE FACTORS

Family response to a member with ataque de nervios would most likely be to do what?

RALLY TO SUPPORT RELATIVE BY REMOVING STRESSORS

A rating instrument is said to have high reliability when which of the following properties is demonstrated: (3x)

RATINGS ARE SIMILAR AMONG DIFFERENT RATERS

Winnicott defined his concept of holding environment as (2x)

SAFE CONTEXT PROVIDED BY CONSISTENT AND RELIABLE PARENTING

Check fasting lipid panels before starting treatment before atypical antipsychotics. Which health quality is implemented according to the aims of the Institute of Medicine?

SAFETY

C&L psych sees 58 yo widowed pt medicine wants to discharge d/t neg workup for abd pain and nausea. Pt refuses to return home to adult kids because the food they prepare is making her sick and causing her sxs. MSE unremarkable and no past psych hx. Next step?

SCHEDULE MEETING WITH THE MEDICAL TEAM, PATIENT, AND FAMILY

American psychiatrists over diagnose in African American patients what mood/psychotic sx?

SCHIZOPHRENIA

"A cross sectional study shows less depressive disorders in people who use a natural pill. They then observe people from the same population for a year and find that prevalence of depression is lower in those who regularly used the pill. They then recruit a sample from the same population and randomly assign people to pill or placebo for one year. This didn't show any difference. Assuming no major flaws in design or analysis, what is the most likely explanation for the discrepancies in results?"

SOME UNKNOWN VARIABLE CONFOUNDED THE NON-RANDOM RESULTS

22 yo M pt complains of fatigue, abd pain, weight loss and believes it is caused by nocturnal emissions of semen. What is pt's ethnic background?

SOUTHEAST ASIAN

A new screening test correctly classifies early mild impairment in 75% and correctly classifies normal mentation in 85%. What does 85% refer to?

SPECIFICITY

In a session, a couple seems blissfully content when happy but enraged when frustrated by the other. They alternate between over- idealizing and devaluing the other. This is an example of?

SPLITTING

What terms best defines the degree of spread of scores about the mean?

STANDARD DEVIATION

An internal medicine resident who is asked by the residency program to take a leave of absence due to erratic behavior must undergo a psychiatric assessment before returning to clinical duties. Which of the following statements best describes the obligation of the examining psychiatrist?

STATE CLEARLY TO PT PRIOR TO EVALUATION THE PURPOSE OF THE EVALUATION AND THAT THE INFORMATION MAY BE TRANSMITTED TO THE RESIDENCY PROGRAM

In evaluating the sanity of a criminal defendant, the forensic psychiatrist should focus on current state of mind, or state of mind at time of offense?

STATE OF MIND AT TIME OF OFFENSE

Spouse of new patient wants to speak with psychiatrist before the appointment. Most appropriate response?

STATE POLITELY AND FIRMLY THAT USUAL PRACTICE IS TO START BY MEETING PATIENT INDIVIDUALLY

A child is jealous of the older sibling's accomplishment and becomes less jealous when he (the patient) accomplishes something. Which defense mechanism is this?

SUBLIMATION

Per psychoanalytic theory, unacceptable affects and impulses are commonly gratified in socially acceptable ways through what

SUBLIMATION

Psychodynamic defense mechanism where jealous kid becomes less jealous by focusing on another outlet

SUBLIMATION

What condition has a higher condition of confidentiality by federal law?

SUBSTANCE USE DISORDERS

Criterion for a covered disability under Americans with Disabilities Act

SUBSTANTIALLY LIMITS ONE OR MORE MAJOR LIFE ACTIVITIES

What psychiatric symptom does not require pharmacologic treatment in the ER?

SUICIDALITY

Most common triggering factor for malpractice claims in psychiatric care

SUICIDE

Lawsuits against psychiatrists most frequently arise from issues concerning

SUICIDE ATTEMPTS

24 yo pt with tetraplegia after MVA showing no signs of grief or acknowledgement of poor prognosis, no significant depressive/anxious Sx. What psych intervention preferred?

SUPPORT BUT DO NOT CONFRONT THE PT ABOUT THE MEDICAL REALITIES

Management of patient in denial immediately after MI

SUPPORTING THE PATIENT, UNLESS DENIAL INTERFERES WITH CARE

What reflects opinion of APA on practice of charging a pt for a missed appointment (3x)

SUPPORTS IT IF IT IS CONTRACTED WITH THE PATIENT

A parent who just learned that her child has been injured and taken to the hospital arranges for a neighbor to care for her other children and then rushed to the hospital. What defense mechanism did the parent use to handle her own fear? (2x)

SUPPRESSION

Semiconsciously diverting attention from a conflict in order to minimize discomfort is an example of what defense mechanism? (3x)

SUPPRESSION

Analytic method that best examines time to remission of symptoms:

SURVIVAL ANALYSIS

Psychiatrist determines that fully informing a pt of tx risks would negatively impact pt's healthͬwelfare Θ documents rationale for tx the patient without informing the patient of all the risks. This psychiatrist is intending to use which of the following?

THERAPEUTIC PRIVILEGE

Rogers v. Commissioner (1983) is a legal case that addressed the right to refuse antipsychotic meds in non-emergency situations. Based on this ruling, pts previously adjudicated as incompetent may not refuse meds in which situation?

THERE IS A COURT-APPROVED SUBSTITUTED JUDGMENT TREATMENT PLAN

What characterizes the majority of defendants with psychiatric disorders who are found unfit to stand trial?

THEY CAN REGAIN FITNESS IN LESS THAN 90 DAYS

Which of the following is the principal service provided by the liaison component of consultation-liaison psychiatry?

TEACHING PSYCHOSOCIAL ASPECTS OF MEDICAL CARE TO OTHER HEALTHCARE WORKERS

Psychiatrist provides psychotherapy to 22 yo pt who lives w/ parents and dx w/ anxiety and depression. Pt reports the father threw 13 yo sibling across the room. Sibling was not injured. Multiple similar episodes. What is the appropriate response for the psychiatrist?

TELL PT THAT THE PSYCHIATRIST MUST REPORT THE ABUSE TO CHILD PROTECTIVE SERVICES

Activity level, regularity, approach-withdrawal to new situations, adaptability, persistence are examples of what

TEMPERAMENTAL VARIABLES

Stage of disease prevention when a burned out caregiver attends a caregiver support group

TERTIARY

Referring patients with schizophrenia to vocational rehabilitation program is what type of prevention

TERTIARY PREVENTION

Absent of gross negligence, dereliction of duty in a malpractice trial is typically established by which of the following?

TESTIMONY OF EXPERT WITNESS

If a third party is supervising a therapy case, the resident should inform the patient about what

THAT HIS CASE WILL BE DISCUSSED

Child custody determinations in most jurisdictions are made using what principle? (4x)

THE BEST INTEREST OF THE CHILD

Scientific development that made NIMH epidemiological catchments area studies possible (9x)

THE DEVELOPMENT OF OPERATIONALIZED DIAGNOSTIC CRITERIA AND STANDARDIZED INSTRUMENTS

Pts with unipolar depression differ epidemiologically from pts with bipolar d/o in what way?

THE DISCREPANCY BETWEEN MALE AND FEMALE LIFETIME PREVALENCE RATES IS GREATER

Relative risk of 1 indicates what about risk of developing the disease of interest?

THE FACTOR HAS NO ASSOCIATION WITH RISK

Best adaptational outcome for immigrant youth

THE FORMATION OF BICULTURAL IDENTITY

Kohut's theory of personality is based on?

THE INDIVIDUAL'S NEED FOR EMPATHIC INTERACTION WITH SELF- OBJECTS

Kohut's theory of personality is based on

THE INDIVIDUAL'S NEED FOR EMPATHIC INTERACTIONS WITH SELF- OBJECTS

Pt has intractable OCD with profound morbidity and considers brain surgery. The surgeon must disclose this in order to obtain a valid informed consent

THE INFORMATION MOST PATIENTS WOULD WANT TO KNOW

80 yo pt was admitted to a nursing home, an employee of the facility notified the pt about the state's laws on advance directives. During that session, the patient most likely received information about what issue

THE LIVING WILL

A characteristic of the placebo effect?

THE PLACEBO EFFECT IS GREATER WHEN THE PT KNOWS THE DOCTOR

"A resident performs a research study utilizing all of the outpatients with any psychiatric disorder, who were treated during the first 2 months of the resident's outpatient experience. The resident discovers that of the 47 patients diagnosed with any psychiatric disorder, six meet the criteria for OCD. Which of the following statistic best defines this info?"

THE POINT PREVALENCE OF OCD IS 6/47

Which of the following is a component of the psychiatrist's formal cultural formulation of a pt's disorder

THE PT'S EXPLANATION OF THE ILLNESS

In evaluating a patient's competence to consent to a medical treatment, the degree of strictness with which the examining psychiatrist considers the question is most affected by which of the following factors?

THE RISK-BENEFIT RATIO OF THE TREATMENT

Studies in which monkeys are raised in varying degrees of isolation have been important in contributing to what theories of human development?

THE SIGNIFICANCE OF ATTACHMENT

Which of following statements identifies what both traditional healing and modern psychotherapeutic practices may have in common

THE THERAPIST HELPS THE PATIENT EXPERIENCE AN EMERGING SENSE OF LEARNING AND MASTERY OVER THE PROBLEM

The Civil Rights of Institutionalized Persons Act of 1980 created a process for which party to intervene on behalf of institutionalized persons to address a grievance or complaint?

THE US DEPARTMENT OF JUSTICE

"Which should raise psych MD's index of suspicion for malingering in cases of litigation in which financial compensation is at issue?"

VAGUELY DEFINED SYMPTOMS

58 yo patient with breast cancer on the C&L service, recently put on palliative level and develops depression. She is hopeless but denies SI. What is the most appropriate response?

VALIDATE THE PATIENT'S FEELINGS

Test that measures what it is supposed to measure (7x)

VALIDITY

29 yo F has worsening mania after stopping VPA. She wants to get pregnant, but her partner is not sure she's psychologically ready. The outpatient psychiatrist and pt haven't discussed her desire for pregnancy, and she has not asked about tx options during pregnancy. ED psychiatrist documents all the concerns. What ethical principle is the ED psychiatrist applying?

VERACITY

Which is a first rank symptom according to Kurt Schneider?

VOICES ARGUING

Involuntarily hospitalized pt refuses meds after physician explains risks, benefits, and alternatives to treatment. Pt reports understanding. Physician then tells patient he will advocate for speedy discharge if pt takes meds. What feature of informed consent is missing?

VOLUNTARINESS

An MS patient is in a study to evaluate a novel MS treatment. If the patient's spouse is requested to participate as a control, what feature of informed consent is being violated?

VOLUNTARISM

Pt consumed a large amount of alcohol and discharged a firearm, wounding a neighbor. Pt remembers nothing of the incident. What is true about the pt's criminal responsibility per the insanity defense?

VOLUNTARY INTOXICATION DOES NOT EXCUSE THE CRIMINAL BEHAVIOR

Socioeconomic status is correlated to risk for mental illness, such as those who are disadvantaged by social class are more likely to develop a mental d/o. Research shows that this correlation is accounted for mainly by greater:

VULNERABILITY TO THE IMPACT OF STRESSFUL LIFE

The reliability of an assessment instrument, such as diagnostic interview, refers to the ability of an instrument to:

YIELD CONSISTENT RESULTS WHEN USED BY DIFFERENT EXAMINERS OR AT DIFFERENT TIMES

͞Reliability͟ of an assessment tool refers to what?

YIELD CONSISTENT RESULTS WHEN USED BY DIFFERENT EXAMINERS OR AT DIFFERENT TIMES

What is used to report the deviation of a value from its group mean, expressed in standard deviation units? (2x)

Z SCORE

Which statement demonstrates that a patient is incompetent to stand trial?

͞I AM A WIZARD AND THEREFORE HAVE DIPLOMATIC IMMUNITY͟

How should the physician respond to a frustrated caregiver who is thinking of separating from her support system (church group)?

͞IT IS IMPORTANT THAT YOU CONTINUE GOING TO YOUR CHURCH GROUP͟

"Consultant evaluates patient for capacity. Patient is refusing meds, procedures, yells at nurses, orders take out into hospital, threatens lawsuits if demands are not met. Patient accuses doctor of being abusive, screams at doctor, when asked for explanation of behavior ͞you're just another person here to abuse me!͟ Best response and advice for the medical team is?"

͞THE PATIENT IS A DIFFICULT PATIENT TO WORK WITH. LET'S TALK ABOUT WAYS TO MANAGE THE PATIENT'S BEHAVIOR.͟


Kaugnay na mga set ng pag-aaral

Reinforcing Reinforcement Theory

View Set

Exam #3 Preview #2 & #3 combined

View Set

CH. 6 TEST CANADA/ GEOGRAPHY/ 7TH GRADE

View Set

226 PrepU - Chapter 24: Asepsis and Infection Control

View Set

Earch Science - Chapter 13 - Mrs. Mann

View Set

Accounting Chapter 13 True or False

View Set

Brain Theories (Whole, Brain Dominance, Triune, Multiple Intelligence, Learning Styles)

View Set

adult development (test 3/final)

View Set